Onko suhteellisuusteoria kumottu?

Anonyymi-ap

Vaikka Albert Einsteinin suhteellisuusteorian on havaittu antavan suunnilleen oikeanlaisia ennusteita, niin onko teorian keskeistä suhteellisuusperiaatetta tarkasteltu äärimmäisissä tapauksissa? Saattanee näet hyvin olla, että suhteellisuusteorian väitetään olevan yleispätevämpi kuin se todellisuudessa on. Filosofi Juha Himanka on ansiokkaasti huomioinut tämän ristiriidan seuraavalla tavalla: "Nobel-palkittu natsifyysikko Philipp Lenard haastoi Einsteinin maan pyörimisliikkeestä akselinsa ympäri, jonka teorian mukaan on oltava suhteellista. Foucaultin heiluri ja vastaavat kokeelliset todistuksen maan kiertoliikkeelle voidaan kyllä selittää myös toisin kuin maan liikkeellä, mutta tällöin – muun universumin kiertäessä maata – kaukaisten tähtien olisi ylitettävä valon nopeus." (3.5.2022 Juha Himanka: Suhteellisuusteoria ideologiana, https://www.luonnonfilosofia.fi/esitelmia/.)

Suhteellisuusteorian sisäinen ristiriita piilee siis siinä, että jos kaukainen tähti kiertää maapallon 24 tunnissa, niin sen ratanopeuden tulee ylittää monin kerroin valonnopeuden, mikä olisi vastoin teorian oletusta valonnopeudesta kosmisena rajanopeutena. Näin ollen Einsteinin suhteellisuusteoria lupaa enemmän kuin se pystyy lunastamaan, ja on suuri mysteeri, miksi tiedemaailma edelleen pitää sitä kiveen hakattuna paradigmana. Vallitseva tilanne muistuttaa ennemmin ideologiaa tai uskontoa kuin itselleen rehellistä tieteellistä maailmankuvaa.

"Pidän enemmän kysymyksistä joihin en osaa vastata kuin vastauksista joita en voi kyseenalaistaa." - Richard Feynman

58

617

    Vastaukset

    Anonyymi (Kirjaudu / Rekisteröidy)
    5000
    • Anonyymi

      Maakeskeinen ajattelu jäi historiaan jo Kopernikuksen aikaan. Sellaisen kaivaminen suhteellisuusteorian "kumoamiseksi" on varsin hyödytöntä. Varsinkin kun meillä on havainto Maan pyörimisliukkeestä avaruudessa sijaitsevien havaintovälineiden avulla.

    • Anonyymi

      Himangan esiintyminen fyysikkona oli melkoinen mahalasku.

    • Anonyymi

      "Suhteellisuusteorian sisäinen ristiriita piilee siis siinä, että jos kaukainen tähti kiertää maapallon 24 tunnissa, niin sen ratanopeuden tulee ylittää monin kerroin valonnopeuden, mikä olisi vastoin teorian oletusta valonnopeudesta kosmisena rajanopeutena."

      Mutta kun se ei kierrä. Samanlaisia näennäisiä ylivalonnopeuksia voisi esittää ison nipun. Mitä Himanka mahtoi tuolla edes tarkoittaa?

      • Anonyymi

        Koska suhteellisuusteorian mukaan liike on suhteellista ja molemmat havaitsijat ovat yhtä oikeassa, niin maapallon pyörähtäminen ympäri, tähden ollessa levossa, on yhtä todellista kuin jos maapallo olisi levossa ja tähti kiertäisi maapallon ympäri. Koska sellainen ajatuskoes johtaa ristiriitaan valonnopeuden kanssa, niin suhteellisuusteoria on joko osin tai kokonaan todettava virheelliseksi.

        Syystä tai toisesta fyysikot eivät halua tästä ristiriidasta keskustella. Siispä tiedon lähettilään tehtävä jää tässäkin tapauksessa filosofeille. En tiedä mikä olisi se paras teoria, mutta suhteellisuusteorian paikka on roskakorissa.


      • Anonyymi
        Anonyymi kirjoitti:

        Koska suhteellisuusteorian mukaan liike on suhteellista ja molemmat havaitsijat ovat yhtä oikeassa, niin maapallon pyörähtäminen ympäri, tähden ollessa levossa, on yhtä todellista kuin jos maapallo olisi levossa ja tähti kiertäisi maapallon ympäri. Koska sellainen ajatuskoes johtaa ristiriitaan valonnopeuden kanssa, niin suhteellisuusteoria on joko osin tai kokonaan todettava virheelliseksi.

        Syystä tai toisesta fyysikot eivät halua tästä ristiriidasta keskustella. Siispä tiedon lähettilään tehtävä jää tässäkin tapauksessa filosofeille. En tiedä mikä olisi se paras teoria, mutta suhteellisuusteorian paikka on roskakorissa.

        Keskustelun puute saattaa johtua siitä, ettei asiassa ole mitään keskustelemista. Jos lyö olkiukkoa, niin eihän sitä kukaan kiellä.


      • Anonyymi
        Anonyymi kirjoitti:

        Koska suhteellisuusteorian mukaan liike on suhteellista ja molemmat havaitsijat ovat yhtä oikeassa, niin maapallon pyörähtäminen ympäri, tähden ollessa levossa, on yhtä todellista kuin jos maapallo olisi levossa ja tähti kiertäisi maapallon ympäri. Koska sellainen ajatuskoes johtaa ristiriitaan valonnopeuden kanssa, niin suhteellisuusteoria on joko osin tai kokonaan todettava virheelliseksi.

        Syystä tai toisesta fyysikot eivät halua tästä ristiriidasta keskustella. Siispä tiedon lähettilään tehtävä jää tässäkin tapauksessa filosofeille. En tiedä mikä olisi se paras teoria, mutta suhteellisuusteorian paikka on roskakorissa.

        Hölmöilet ja rakentelet olkiukkoja. Suhteellisuusteoriassa pyörimisliike on edelleenkin absoluuttista ja ollut sitä alusta asti. Tämä on kokeellisesti varmistettu jo vuonna 1913 eli jo 111 vuotta sitten.

        https://en.wikipedia.org/wiki/Sagnac_effect

        Kaupasta saa optisia gyroskooppeja joiden toiminta perustuu tuohon ilmiöön. Vuonna 1926 mitattiin Maapallon pyörimisliike käyttäen tuota ilmiötä.

        https://en.wikipedia.org/wiki/Michelson-Gale-Pearson_experiment

        Jos aiot kaataa suhteellisuusteorian niin ota nyt ensin selvää mitä se suhteellisuusteoria väittää. Muuten kukaan ei viitsi vastailla viesteihisi aiheesta. Kuten Wikipedian artikkelissa todetaan:

        "The Sagnac effect has stimulated a century long debate on its meaning and interpretation,[29][30][31] much of this debate being surprising since the effect is perfectly well understood in the context of special relativity. "


      • Anonyymi
        Anonyymi kirjoitti:

        Koska suhteellisuusteorian mukaan liike on suhteellista ja molemmat havaitsijat ovat yhtä oikeassa, niin maapallon pyörähtäminen ympäri, tähden ollessa levossa, on yhtä todellista kuin jos maapallo olisi levossa ja tähti kiertäisi maapallon ympäri. Koska sellainen ajatuskoes johtaa ristiriitaan valonnopeuden kanssa, niin suhteellisuusteoria on joko osin tai kokonaan todettava virheelliseksi.

        Syystä tai toisesta fyysikot eivät halua tästä ristiriidasta keskustella. Siispä tiedon lähettilään tehtävä jää tässäkin tapauksessa filosofeille. En tiedä mikä olisi se paras teoria, mutta suhteellisuusteorian paikka on roskakorissa.

        Tähti avaruudessa ja maapallo ovat samassa inertiaalikoordinaatistossa. Silloin niillä ei ole keskinäistä kiihtyvää liikettä, joka voisi vaikuttaa valon nopeuden mittaamiseen. Ts. molemmat tällaiset havaitisijat toteaisivat valonnopeuden vakioiseksi.

        Mainitun ilmiön vaikutusta voidaan hyödyntää gyroskooppien rakentamisessa, jolloin ei tarvita mekaanisia osia laitteeseen, vaan voidaan hyödyntää valonnopeuden mittaamista (laserin avulla). Tätä ilmiötä kutsutaan Sagnac effektiksi. Sitä hyödyntää esim. GPS-satelliiteissa niiden kellojen synkronoimisessa silloin, kun satelliiteissa on otettava huomioon maapallon pyörimisliike niiden suhteen.


    • Anonyymi

      Onkohan Himanka koskaan kuullute sellaisesta kuin lyödään olkiukkoa? Tuli vain mieleen tuosta ulostulosta.

    • Himanka on hyvässä seurassa: muistan jo opiskeluaikoinani kiemurrelleeni myötähäpeästä lukiessani, kuinka mm. Suuret Mestarit Wittgenstein ja Schopenhauer kumpikin tahollaan häpäisivät itsensä fundeeraamalla luonnontieteellisiä ilmiöitä, jotka jo silloiset fyysikot olivat mitanneet, tutkineet ja selittäneet ainakin oikeansuuntaisesti, senaikaisen tiedon pohjalta.

      "Suutari pysyköön lestissään!"

    • Anonyymi

      Tiedeohjelmassa väitettiin, että ilman kuuta maapallo "muljahtelisi" satunnaisesti ja vuodenajat menisivät ihan sekaisin. Onkohan tuo totta.

      • Anonyymi

        Pitää paikkansa. Mutta "muljahtelu" tapahtuu tuhansien vuosien kuluessa. Kuu vakauttaa pallon pyörimistä, varsinkin kun kuu on poikkeuksellisen suuri kiertolainen planeetalle.


      • Anonyymi

        Kyllä tuo totta taitaa olla, sillä radat ovat tietyllä tapaa lukittautuneita toisiinsa. Kuun vaikutus on riittävä, jotta ratataso säilyy. En äkkisältään osaa laskea "muljahduksen" jaksonaikaa, mutta sanoisin että se on tuhansia(miljoonia?) vuosia, koska kuun voimavaikutus on sen verran pieni. Tarvittaisiin siis joku kappale, joka tönäisisi maapallon nurin. Auringon aikaansaama presessioliike maapallo-hyrrässä on muistaakseni 23 000 vuotta.
        Jopa lähiplaneetat muuttavat olosuhteita enemmän maapallolla kuin kuun painovoima.


    • Anonyymi

      Syksy Räsänen kehoitti blogissaan seuraamaan aeon.co kosmologian uutisia. Siellä on useitakin artikkeleita pimeän aineen vaihtoehdosta. Tässä esim. New Yorkin yliopiston fysiikan professori selittää MOND-teoriasta, miten pimeää ainetta ei todennäköisesti ole olemassa vaan MOND on paljon parempi selitys:
      https://aeon.co/essays/we-should-explore-alternatives-to-the-standard-model-of-cosmology
      Pimeää ainetta + suhtista voidaan siis pitää falsifioituna, eivät päde galaksien mittakaavassa.

      • Anonyymi

        "Pimeää ainetta + suhtista voidaan siis pitää falsifioituna, eivät päde galaksien mittakaavassa."

        Ei ole. Sinulla on erikoinen käsitys fysiikasta. Yksi todeksi osoittamaton teoria (MOND) ei kumoa moneen kertaan kokeellisesti vahvistettua teoriaa (GR). Ja kuten itse kirjoitit, pimeäle aineelle on useita (muitakin kuin MOND) vaihtoehtoja, joista ainuttakaan ei ole vielä todistettu.


      • Anonyymi
        Anonyymi kirjoitti:

        "Pimeää ainetta suhtista voidaan siis pitää falsifioituna, eivät päde galaksien mittakaavassa."

        Ei ole. Sinulla on erikoinen käsitys fysiikasta. Yksi todeksi osoittamaton teoria (MOND) ei kumoa moneen kertaan kokeellisesti vahvistettua teoriaa (GR). Ja kuten itse kirjoitit, pimeäle aineelle on useita (muitakin kuin MOND) vaihtoehtoja, joista ainuttakaan ei ole vielä todistettu.

        Jos suhtis olisi falsifioitu, niin siitä olisi revitetty kaikkien tiedelehtien lisäksi myös Seiskassa ja MV-lehdessä maailman kaikkien muiden medioiden lisäksi.


      • Anonyymi
        Anonyymi kirjoitti:

        "Pimeää ainetta suhtista voidaan siis pitää falsifioituna, eivät päde galaksien mittakaavassa."

        Ei ole. Sinulla on erikoinen käsitys fysiikasta. Yksi todeksi osoittamaton teoria (MOND) ei kumoa moneen kertaan kokeellisesti vahvistettua teoriaa (GR). Ja kuten itse kirjoitit, pimeäle aineelle on useita (muitakin kuin MOND) vaihtoehtoja, joista ainuttakaan ei ole vielä todistettu.

        "Ei ole. Sinulla on erikoinen käsitys fysiikasta. Yksi todeksi osoittamaton teoria (MOND) ei kumoa moneen kertaan kokeellisesti vahvistettua teoriaa (GR)."

        GR on nimenomaan moneen kertaan kokeellisesti falsifioitu galaksien pyörimistutkimuksilla. Sen sijaan ns. pimeästä aineesta ei ole ainoataan havaintoa, eikä edes teoreettista viitekehystä, mitä se voisi mahdollisesti olla.


    • Anonyymi

      On kumottu terv.Plejadi Harmonia

    • GR ei sovi galakseja suurempaan maailmaan, kaikkeuteen. Sen pohjalta saadaan galaksitutkimuksessa väärät tulokset, varsinkin kun se yhdistetään BB teoriaan. Jotain pitää muuttaa.

      Ilmeisesti koordinaatisto 3Dstä 4Dhen. Aika ei ole ulottuvuus. Tilaulottuvuuksia vaan on neljä kaikkeuden kohdalla. Sillä tavalla saadaan kuvattua kaikkeuden ulkopuolettomuus, mikä on looginen tosiasia.

      Ja painovoima on todellinen voima. Sen vaikutus tunnetaan, muttei tiedetä mitä se on ja mikä sitä välittää. GR ajattelu estää painovoiman tutkimisen.

      Painovoiman kaavan muutoksella päästään pimeästä aineesta. MOND.

      • Anonyymi

        Paskaa jauhamalla on helppo kumota mikä tahansa suhteellisuusteoria.


      • Anonyymi

        MOND ei ole vielä kuin hypoteesi. Eikä se kumoa BB teoriaa, kuten olet väittänyt.
        MOND hypoteesilla ei ole juurikaan empiiristä näyttöä, kun sen sijaan suhteellisuusteorialla on vaikka miten paljon. Väite, ettei suhteellisuusteoria pätisi galaksien välisillä etäisyyksillä on näyttämättä toteen.


      • Anonyymi kirjoitti:

        Paskaa jauhamalla on helppo kumota mikä tahansa suhteellisuusteoria.

        Kuin niin? Tiede on teorioita, jotka vaihtuvat ja kehittyvät. Miten sinä kehittäisit kosmologian teoriaa?


      • Anonyymi kirjoitti:

        MOND ei ole vielä kuin hypoteesi. Eikä se kumoa BB teoriaa, kuten olet väittänyt.
        MOND hypoteesilla ei ole juurikaan empiiristä näyttöä, kun sen sijaan suhteellisuusteorialla on vaikka miten paljon. Väite, ettei suhteellisuusteoria pätisi galaksien välisillä etäisyyksillä on näyttämättä toteen.

        Kyllä se kumoaa BBn ja on oikea asenne tieteeseen.


      • Anonyymi
        santtunen kirjoitti:

        Kyllä se kumoaa BBn ja on oikea asenne tieteeseen.

        Miten ihmeessä MOND kumoaa BB teorian? Ei ainakaan MOND luojan, eikä kenenkään muunkaan kosmologin mielestä.

        Miten minä kehittäisin kosmologian teoriaa?
        En mitenkään. Vaikka tiedän aiheesta huomattavasti keskivertokansalaista enemmän, niin tiedän, ettei minulla ole minkäänlaista kompetenssia kehittää kosmologian teoriaa. Sellaiset 6 - 8 vuotta alan yliopistokoulutusta puuttuu.

        Entä mikä meidän eromme on? Minä ymmärrän rajoitukseni, sinä et. Tiedän, ettei minulla ole kosmologiabn tutkimukseen mitään annettavaa, vaikka tietoni alalta ylittävät sinun tietosi monin verroin.


      • Anonyymi
        Anonyymi kirjoitti:

        Miten ihmeessä MOND kumoaa BB teorian? Ei ainakaan MOND luojan, eikä kenenkään muunkaan kosmologin mielestä.

        Miten minä kehittäisin kosmologian teoriaa?
        En mitenkään. Vaikka tiedän aiheesta huomattavasti keskivertokansalaista enemmän, niin tiedän, ettei minulla ole minkäänlaista kompetenssia kehittää kosmologian teoriaa. Sellaiset 6 - 8 vuotta alan yliopistokoulutusta puuttuu.

        Entä mikä meidän eromme on? Minä ymmärrän rajoitukseni, sinä et. Tiedän, ettei minulla ole kosmologiabn tutkimukseen mitään annettavaa, vaikka tietoni alalta ylittävät sinun tietosi monin verroin.

        Ylivertaisuusharhaa ei siitä kärsivä omassa itsessään tunnista. Nähty niin moneen kertaan aph:n ja nyt sant... kohdalla.


      • Anonyymi kirjoitti:

        Miten ihmeessä MOND kumoaa BB teorian? Ei ainakaan MOND luojan, eikä kenenkään muunkaan kosmologin mielestä.

        Miten minä kehittäisin kosmologian teoriaa?
        En mitenkään. Vaikka tiedän aiheesta huomattavasti keskivertokansalaista enemmän, niin tiedän, ettei minulla ole minkäänlaista kompetenssia kehittää kosmologian teoriaa. Sellaiset 6 - 8 vuotta alan yliopistokoulutusta puuttuu.

        Entä mikä meidän eromme on? Minä ymmärrän rajoitukseni, sinä et. Tiedän, ettei minulla ole kosmologiabn tutkimukseen mitään annettavaa, vaikka tietoni alalta ylittävät sinun tietosi monin verroin.

        Minä olen tutkinut kosmologiaa 60 v ja nyt Hubblen ja Webbin kuvia, ja artikkeleita, niissä on objektiiviset havainnot galakseista ja BBn kannattajat yrittävät sivuuttaa niiden selvät tulokset.


      • Anonyymi
        santtunen kirjoitti:

        Minä olen tutkinut kosmologiaa 60 v ja nyt Hubblen ja Webbin kuvia, ja artikkeleita, niissä on objektiiviset havainnot galakseista ja BBn kannattajat yrittävät sivuuttaa niiden selvät tulokset.

        Tutkinut kosmologiaa? Millä ihmeen eväillä? Et kai luule, että netissä ja lehdissä julkaistujen Hubble kuvien katselu on kosmologian tutkimusta?
        Jos olet lukenut myös tieteellisiä artikkeleja noita kuvia koskien, niin voit varmasti linkittää pari, joissa BB teoriaa pidetään kumottuna.

        Ei kuvien katseleminen ja toisten tekstien lukeminen ole kosmologian tutkimusta. Sitten kun sinulla on alalta vähintää gradu ja väitöskirjatutkimus meneillään, voit ottaa jotain kantaa asiaan.


      • Anonyymi kirjoitti:

        Tutkinut kosmologiaa? Millä ihmeen eväillä? Et kai luule, että netissä ja lehdissä julkaistujen Hubble kuvien katselu on kosmologian tutkimusta?
        Jos olet lukenut myös tieteellisiä artikkeleja noita kuvia koskien, niin voit varmasti linkittää pari, joissa BB teoriaa pidetään kumottuna.

        Ei kuvien katseleminen ja toisten tekstien lukeminen ole kosmologian tutkimusta. Sitten kun sinulla on alalta vähintää gradu ja väitöskirjatutkimus meneillään, voit ottaa jotain kantaa asiaan.

        Fil.kand eväillä. Psykologia, filosofia. Tähtitieteen oppikirjat. Tietokirjailija, filosofin eväillä. Jos olen oikeassa, niin turha kysyä eväitä. Pitää osoittaa, että olen väärässä.

        BBhen vetoaminen ei käy. Se on argumenttivirhe, kun kerran olen mielestäni kumonnut sen, olen sen kanssa eri mieltä.

        Eikä kompetenssiin. Sellaisia tukijoita on, joilla on kompetenssi ja jotka ovat samaa mieltä BBstä. Eric Lerner, Louis Marmet jne. ACG ja MOND.


      • Anonyymi
        santtunen kirjoitti:

        Fil.kand eväillä. Psykologia, filosofia. Tähtitieteen oppikirjat. Tietokirjailija, filosofin eväillä. Jos olen oikeassa, niin turha kysyä eväitä. Pitää osoittaa, että olen väärässä.

        BBhen vetoaminen ei käy. Se on argumenttivirhe, kun kerran olen mielestäni kumonnut sen, olen sen kanssa eri mieltä.

        Eikä kompetenssiin. Sellaisia tukijoita on, joilla on kompetenssi ja jotka ovat samaa mieltä BBstä. Eric Lerner, Louis Marmet jne. ACG ja MOND.

        "Pitää osoittaa, että olen väärässä."
        "BBhen vetoaminen ei käy. Se on argumenttivirhe, kun kerran olen mielestäni kumonnut sen, olen sen kanssa eri mieltä."

        Että näillä eväillä. Voi pyhä jeesus sentään :D. Olet niitä hemmoja, jotka ovat niin tyhmiä, etteivät tajua miten tyhmiä ovat. Ei pahalla, mutta ei juuri hyvälläkään.


      • Anonyymi
        santtunen kirjoitti:

        Fil.kand eväillä. Psykologia, filosofia. Tähtitieteen oppikirjat. Tietokirjailija, filosofin eväillä. Jos olen oikeassa, niin turha kysyä eväitä. Pitää osoittaa, että olen väärässä.

        BBhen vetoaminen ei käy. Se on argumenttivirhe, kun kerran olen mielestäni kumonnut sen, olen sen kanssa eri mieltä.

        Eikä kompetenssiin. Sellaisia tukijoita on, joilla on kompetenssi ja jotka ovat samaa mieltä BBstä. Eric Lerner, Louis Marmet jne. ACG ja MOND.

        Eli täysin alalle kouluttamaton, jolla ei ole muuta perustetta kuin MuTu.
        Ei noilla eväillä tiedettä tehdä millään luonnontieteen alalla.

        Kerro ne tieteelliset perusteet, millä olet BB:n kumonnut. Hubblen kuvien katselu ai satavarmasti riitä, eikä myöskään filosofian kandin koulutus.
        Juttujesi perusteella minulle on syntynyt kuva, ettet osaa edes modernin fysikkan perusteita, joita kosmologian edes jonkinasteinen ymmärtäminen vaatisi. Jutut useista alkuräjähdyksistä näkyvässä avaruudessa osoittavat sen jo selvästi, muista huuhailuistasi puhumattakaan.

        Jos ymmärrät jotain alkuräjähdysteoriasta, osaat varmasti kertoa, miksi alkuräjähdyksiä ei ole voinut olla tuntemassamme maailmankaikkeudessa useita. Minäkin sen osaisin kertoa kvanttifysiikan lakeihin nojautuen. Mutta minä en sentään kehtaisi ikinä väittää olevani kosmologian tai fysiikan tutkija.


      • Anonyymi kirjoitti:

        "Pitää osoittaa, että olen väärässä."
        "BBhen vetoaminen ei käy. Se on argumenttivirhe, kun kerran olen mielestäni kumonnut sen, olen sen kanssa eri mieltä."

        Että näillä eväillä. Voi pyhä jeesus sentään :D. Olet niitä hemmoja, jotka ovat niin tyhmiä, etteivät tajua miten tyhmiä ovat. Ei pahalla, mutta ei juuri hyvälläkään.

        Mitätöinti ei ole argumentti. Se että uskot auktoriteettiuskoisesti BB teoriaan ja naurat muille, on epätieteellistä. Jos olisit vähäkin tieteellinen, pystyisit sanomaan mikä ajatuksessani on naurettavaa, mutta et pysty.

        Tietysti on naurettavaa olla koko tiedeyhteisöä vastaan. Mutta se vaan näyttää siltä. Osa tiedeyhteisöstä on tällä puolella. BBn kannattajille se on todella karvas ja mahdotln pala myöntää.

        Jos myöntävät, BBn valta on ohi.

        Se on vaan yksi koulukunta tieteessä silloin.


      • Anonyymi kirjoitti:

        Eli täysin alalle kouluttamaton, jolla ei ole muuta perustetta kuin MuTu.
        Ei noilla eväillä tiedettä tehdä millään luonnontieteen alalla.

        Kerro ne tieteelliset perusteet, millä olet BB:n kumonnut. Hubblen kuvien katselu ai satavarmasti riitä, eikä myöskään filosofian kandin koulutus.
        Juttujesi perusteella minulle on syntynyt kuva, ettet osaa edes modernin fysikkan perusteita, joita kosmologian edes jonkinasteinen ymmärtäminen vaatisi. Jutut useista alkuräjähdyksistä näkyvässä avaruudessa osoittavat sen jo selvästi, muista huuhailuistasi puhumattakaan.

        Jos ymmärrät jotain alkuräjähdysteoriasta, osaat varmasti kertoa, miksi alkuräjähdyksiä ei ole voinut olla tuntemassamme maailmankaikkeudessa useita. Minäkin sen osaisin kertoa kvanttifysiikan lakeihin nojautuen. Mutta minä en sentään kehtaisi ikinä väittää olevani kosmologian tai fysiikan tutkija.

        Kosmologia ei ole luonnontiede pelkästään, vaan ennen kaikkea tähtitiedettä ja filosofiaa yhtä aikaa. Filosofiaa enemmän kuin pelkkää fysiikkaa. Metafysiikkaa, ei pelkkää fysiikkaa.

        Tieteenä. Lisäksi se on maailmankatsomusta ja uskontoa: Millainen universumi on?

        Tuntemamme maailmankaikkeus on subjektiivisen käsite. Objektiivisesti, todellisuudessa, tutkimme koko universumia, kaikkeutta. Se on eri asia kuin paikallinen osauniversumi. BB sekoittaa nämä todellisuudessa olevat kaksi eri asiaa, ja väittää, ettei osauniversumia ole.

        On se, ja se alkaa, ei kaikkeus.

        Kun osaan sanoa tämän, ja tämä on totta, 60 v opiskeluni ei ole mennyt hukkaan. Ja kas kummaa, minulla on kompetenssi meriiteistä riippumatta. Parempi kompetenssi kuin BB matematiikan fakki-idiooteilla, jotka nykyään vain kosmologeiksi lasketaan.


      • Anonyymi
        santtunen kirjoitti:

        Kosmologia ei ole luonnontiede pelkästään, vaan ennen kaikkea tähtitiedettä ja filosofiaa yhtä aikaa. Filosofiaa enemmän kuin pelkkää fysiikkaa. Metafysiikkaa, ei pelkkää fysiikkaa.

        Tieteenä. Lisäksi se on maailmankatsomusta ja uskontoa: Millainen universumi on?

        Tuntemamme maailmankaikkeus on subjektiivisen käsite. Objektiivisesti, todellisuudessa, tutkimme koko universumia, kaikkeutta. Se on eri asia kuin paikallinen osauniversumi. BB sekoittaa nämä todellisuudessa olevat kaksi eri asiaa, ja väittää, ettei osauniversumia ole.

        On se, ja se alkaa, ei kaikkeus.

        Kun osaan sanoa tämän, ja tämä on totta, 60 v opiskeluni ei ole mennyt hukkaan. Ja kas kummaa, minulla on kompetenssi meriiteistä riippumatta. Parempi kompetenssi kuin BB matematiikan fakki-idiooteilla, jotka nykyään vain kosmologeiksi lasketaan.

        "Kosmologia ei ole luonnontiede pelkästään, vaan ennen kaikkea tähtitiedettä ja filosofiaa yhtä aikaa."

        Idioottimainen paskanjauhanta ei ole kumpaakaan.

        "Ja kas kummaa, minulla on kompetenssi meriiteistä riippumatta. "

        Sinulla on todellakin kompetenssi idioottimaiseen paskanjauhantaan.


      • Anonyymi kirjoitti:

        "Kosmologia ei ole luonnontiede pelkästään, vaan ennen kaikkea tähtitiedettä ja filosofiaa yhtä aikaa."

        Idioottimainen paskanjauhanta ei ole kumpaakaan.

        "Ja kas kummaa, minulla on kompetenssi meriiteistä riippumatta. "

        Sinulla on todellakin kompetenssi idioottimaiseen paskanjauhantaan.

        Ihmiset luulevat, että heillä on kompetenssi kun ovat lukeneet oppikirjat ja ovat sen takia BBn kannalla. Kompetenssi on vasta sitten, kun on lukenut oppikirjat ja BBn vastustajien artikkelit ja tehnyt omat arvionsa ja lukenut artikkeleita Webbin tuloksista. Jos sen jälkeen edelleen on BBn kannalla, niin ei se enää tiedettä ole, vaan sokeaa auktoriteettiuskoa. BB teoria on lahkolaista pseudotiedettä, ei enää tiedettä.

        Diktatorisesti vaan vallassa. Ja kaatuu kuin korttitalo.


      • Anonyymi
        Anonyymi kirjoitti:

        MOND ei ole vielä kuin hypoteesi. Eikä se kumoa BB teoriaa, kuten olet väittänyt.
        MOND hypoteesilla ei ole juurikaan empiiristä näyttöä, kun sen sijaan suhteellisuusteorialla on vaikka miten paljon. Väite, ettei suhteellisuusteoria pätisi galaksien välisillä etäisyyksillä on näyttämättä toteen.

        "MOND ei ole vielä kuin hypoteesi. Eikä se kumoa BB teoriaa, kuten olet väittänyt."

        Mond ei kumoa BB:tä, mutta havainnot pyörivistä galakseista sen sijaan falsifioivat suhtiksen.

        Lisäksi MOND sentään on tieteellinen hypoteesi. Samaa ei voi sanoa pimeästä aineesta.

        Pimeä aine muuten olisi hypoteesi, mutta teoreettinen viitekehys puuttuu, mitä se voisi olla. Kaikki pimeän aineen kandidaatit on kokeellisesti falsifioitu.


    • Anonyymi

      Olisihan se lajissaan huikea tapahtuma, jos Suomi24:n anonyymi kumoaisi suhteellisuusteorian. En taida yöunia menettää sitä odotellessa.

    • Anonyymi

      Filosofi Himanka on jo aiemmin saanut tyrmäysiskuja leukaperiinsä tieteellisessä väittelyssä, eikä voi kuin ihmetellä, miten hän on mm. Enqvistiltä saamansa turpasaunan jälkeen noussut jälleen kehään.

      Katoin tuon Luonnonfilosofian seuran videon ja voinkin saman tien tiputtaa herra filosofin perseelleen seuraavalla vasen-oikea-yhdistelmällä:

      - Lenardin "vastaesimerkki" oli lapsellinen väärinymmärrys eikä siitä sen takia ole tieteen foorumeilla keskusteltu. Asia on näet niin, ettei liikkeen suhteellisuusperiaate ole voimassa, jos liike on pyörivää tai mitä tahansa muuta kuin tasaista ja suoraviivaista.

      - Karl Popperin mielipide kaukovaikutuksesta ja suhteellisuusteoriasta oli tavattoman naiivi, sillä hän ei ymmärtänyt kvanttimekaniikan formalismia. Voidaan näet ihan matemaattisesti osoittaa, ettei kvanttimekaniikan mallissa voi lähettää valoa nopeammin kulkevaa signaalia. Tulos löytyy googlesta: "no-communication theorem".

      • Anonyymi

        Ja mitä tuo Himanka lopussa puhui "samanaikaisuuden ongelmasta"? Se samanaikaisuus on SUHTEELLISTA eikä teoria välitä siitä. Se mistä teoria välittää on tapahtumien kausaalisuus, joka on voimassa kaikissa koordinaatistoissa.


      • Anonyymi
        Anonyymi kirjoitti:

        Ja mitä tuo Himanka lopussa puhui "samanaikaisuuden ongelmasta"? Se samanaikaisuus on SUHTEELLISTA eikä teoria välitä siitä. Se mistä teoria välittää on tapahtumien kausaalisuus, joka on voimassa kaikissa koordinaatistoissa.

        Totta. Samanaikaisuuden suhteellisuus ei ole mikään ongelma (kuten Himanka ilmeisesti väittää), mutta kausaalisuuden suhteellisuus olisi todellinen killeri teorian kannalta. Mutta onneksi suhteellisuusteoriassa on on tapahtumilla aina sama kausaalinen järjestys jokaisen lepokoordinaatiston suhteen: https://i.stack.imgur.com/mefSU.png


    • Anonyymi

      "Suhteellisuusteoriassa pyörimisliike on edelleenkin absoluuttista ja ollut sitä alusta asti."

      Tämä voi olla hyvin sekaannuttava sanomistapa. GR:ssä sanotaan esim. kiihtyvyydestä siten, että ominaiskiihtyvyys on absoluuttista, mutta muunlainen kiihtyvyys ei. Ensimmäinen tarkoittaa sitä, että sillä on yksi ja sama arvo kaikissa koordinaatistoissa. Ja jälkimmäinen on sitä, että esim. erilailla itseäään kiihdyttävien epäinertiaalisten havaitsijoiden mukaan eri kohteet näyttäytyisivät heille eri tavalla kiihdyttäviltä. Tämä kaikki pätee myös pyörimiselle. Vaikka jostain syystä ominaispyörimisnopeus tai proper rotation on erittäin vähän käytetty termi. Ominaisrotaatio olisi määritelty juuri siten, että objekti mittaa luonaan oman pyörimisensä. Mainitsit joitain näistä mittausmenetelmistä. Tämän mittaamisen takia tätä absoluuttisuuden määritelmää voi verrata Newtonin aikaisiin määritelmiin.

      Miksi kuitenkin sanoin, että se sekaannuttaa asioita, on se, että GR:n tarkoitus on sanoa myös, että kaikki suureet, jotka ovat nimeltään nopeus, kiihtyvyys tai pyörimisnopeus ym. ovat kaikki suhteellisia. Ja siinä on paljon tärkeämpää sanoa aina, että kaikki nämä muut asiat kuin ominais-asiat ovat havaitsijasta riippuvaisia. GR merkitsee tässä GR:ssä ensimmäistä kertaa ollutta matemattista esitystä liikkeelle ja koordinaatistoille. Tässä ei tarvitse puhua siitä, onko olemassa massaa, ja massojen välistä gravitaatiota. (Lisäksi jostain välistä pudotetaan pois myös tapaus, missä tätä matematiikkaa sovellettaisiin ilman valonnopeuden rajaa) Ilman tätä GR:n matematiikkaa ei ollut mahdollista sanoa, mitä tarkoittaa, että pyörimisliike on suhteellinen eri pyörijöiden kesken, ja että missä fysikaaliset faktat ovat. Sen sijaan modifioitiin luonnonlakeja tai sitten poistuttiin muunnoksella lopullisesti pois sellaisen havaitsijan näkemistä luvuista, jolle helpoin liikelaki ei ollut voimassa. GR:n työ tässä oli siis myös kertoa jokin liikelaki. Kun se oli, niin tästä yhtenäisestä liikelaista seuraa se, että mistä tahansa koordinaatistosta voidaan tehdä tämän lain lausekkeen muunnos ominaiskoordinaatteihin ja tulos, mitä saatu lauseke antaa on aina sama.

      "Vaikka Albert Einsteinin suhteellisuusteorian on havaittu antavan suunnilleen oikeanlaisia ennusteita."

      Ei kenenkään tule tietää, mitä tarkalleen pitää ennustaa, jos kokeet ovat vain suunnilleen oikeanlaisia kokeita.

      "onko teorian keskeistä suhteellisuusperiaatetta tarkasteltu äärimmäisissä tapauksissa?"

      Ei. On olemassa ainakin kolme erilaista äärimmäisyyttä. Eikä ihminen pysty muodostamaan niistä yhtäkään.

      "Saattanee näet hyvin olla, että suhteellisuusteorian väitetään olevan yleispätevämpi kuin se todellisuudessa on. Filosofi Juha Himanka on ansiokkaasti huomioinut tämän ristiriidan"

      Linkkisi ei toiminut, mutta Himanka ei ole tainnut mainita GR:n päteevyysalueesta yhtään mitään samalla tavalla, kuin miten äärimmäisessä kvanttigravitaatiossa puhutaan GR:n pätevyysalueesta.

      "Nobel-palkittu natsifyysikko Philipp Lenard haastoi Einsteinin maan pyörimisliikkeestä akselinsa ympäri, jonka teorian mukaan on oltava suhteellista. Foucaultin heiluri ja vastaavat kokeelliset todistuksen maan kiertoliikkeelle voidaan kyllä selittää myös toisin kuin maan liikkeellä, mutta tällöin – muun universumin kiertäessä maata – kaukaisten tähtien olisi ylitettävä valon nopeus." (3.5.2022 Juha Himanka: Suhteellisuusteoria ideologiana,"

      Lenardin pyörimisliikekommentti on huono tapa alkaa arvioida Einsteinia, koska hän sanoi sen sellaisessa vaiheessa 1920, kun GR oli jo olemassa ja molemmat keskustelijat luulivat puhuvansa GR:stä. Havaitsijan pyöriminen ei silti mitenkään edellytä gravitaatiota, ja asiasta olisi voitu keskustella jo aiemmin sellaisessa selvittämättömältä tuntuvassa tilassa kuin SR-avaruudessa pyörivä havaitsija (ja tämän metriikka kyseiselle avaruudelle). Asian keskustelu oli lähinnä sitä, että Lenardia informoidaan siitä, että mitä Einsteinin teoria oikeasti sisältää.

      Oletko sinä ja Himanka halunneet puhua Lenardin ongelmasta valonnopeuden kanssa, vai sellaisesta ideasta, että 'entä jos maapallo ei pyöri, mutta koko universumi pyörii sen ympärillä'? Nämä ovat toisiaan paljon pois sulkevia aiheita siten, että toista ei toisen tapauksen voittaessa tarvitse käydä ainakaan väärässä olleelle suhteellisuudelle enää mistään syystä läpi. Lenard ei uskonut pyörivään universumiin tässä yhdessä keskustelussa eikä myöskään sen ulkopuolella?

      Pohdin äskeisessä ikäänkuin, että miksi puhutaan Focault-heilurista ja valonnopesta pyörimisessä yhtäaikaa. Ja arvostelen näytetyn suorituksen siltä kannalta, että sillä yritettiin yhtä noista äskeisistä.

      1

      • Anonyymi

        Focaultin heiluria ei voi selittää ilman gravitaatiota. Jos vaatisi paikalle toimivan Focault-heilurin, niin ei voisi olla varma siitä, että onko oikein arvostella pyöriviä havaitsijoita myös ilman gravitaatiota. Jolloin Einsteinin kritisointi on pienemmällä (kritiikin pätevyyden) alueella tehty kuin, mitä oli mahdollista. Onko se heiluri ollut tässä siksi, että henkilö joka haluaa kommentoida Einsteinia joutuu ensin todistamaan, että pyöriminen on olemassa eikä fiktiivistä (avaruudet ja koordinaatistot on tehty matemaattisesti ja on olemassa joku joka ei usko, että pyöriminen on olemassa vain sen takia, että se niissä esitetään)? Tämän pyörimisen epäfiktiivisyyden voi minusta todistaa sillä, että pörivän henkilön kasvoihin maalataan merkkejä tietyin välein. Ja kun nämä merkit liikkuvat ja toistuvat hänen katselijoidensa silmin nähtynä, niin se on riittävä todiste siitä, että hän pyörii (pyörimismatematiikasta saatujen tulosten mukaan).

        "Suhteellisuusteorian sisäinen ristiriita piilee siis siinä, että jos kaukainen tähti kiertää maapallon 24 tunnissa, niin sen ratanopeuden tulee ylittää monin kerroin valonnopeuden, mikä olisi vastoin teorian oletusta valonnopeudesta kosmisena rajanopeutena. Näin ollen Einsteinin suhteellisuusteoria lupaa enemmän kuin se pystyy lunastamaan, ja on suuri mysteeri, miksi tiedemaailma edelleen pitää sitä kiveen hakattuna paradigmana. Vallitseva tilanne muistuttaa ennemmin ideologiaa tai uskontoa kuin itselleen rehellistä tieteellistä maailmankuvaa."

        Lähellä oleva aurinko kiertää siis jaloillaan pyörivän ihmisen ympäri nopeammin kuin valo. Siitä saa 'hyvin sisäisen' eli matemaattista jargonia koskevan lauseen, että mikä nopeus ja mikä raja on oikeasti mitäkin. Eli pikemminkin kieliopillisia sääntöjä siitä, että mitä lauseita pitää esittää, jotta lauseet eivät olisi ristiriidassa minkään kanssa. Erikoisessa suhteellisuusteoriassa lukee tällaisen korjauksen jälkeen näin:
        https://en.wikipedia.org/wiki/Postulates_of_special_relativity
        'As measured in any inertial frame of reference, light is always propagated in empty space with a definite velocity c that is independent of the state of motion of the emitting body. Or: the speed of light in free space has the same value c in all inertial frames of reference.'

        Tämä on tässä oletus. Joten lause joka sanoo, että jotakin on luvattu, on mahdollisesti ristiriidassa kirjallisten lähteiden kanssa.

        Pyörivän ihmisen näkemä aurinko ei ole määritelmän mukainen inertiaalisessa koordinaatistossa liikkuva objekti. Tutkiessaan pyörivää ihmistä tulisi tehneeksi näin:
        https://physics.stackexchange.com/questions/154834/how-do-i-transform-onto-a-relativistic-rotating-frame-of-reference

        Teoriat olettavat varsinkin GR:ssä sen, että tähden liike on luonnonlakien mukainen vaikka ihmisen pää pyörisi miten tahansa. Koska kyseessä oli fysiikan uusi teoria, se myös lupasi kertoa, mikä on luonnonlaki, jota tässä sitten noudatetaan. Voidaan kuitenkin olla vielä montaa mieltä siitä, että onko luonnonlaki lupaus vai oletus.

        Einsteinin luonnolaki tähdille ja sille, miten myös valon yksittäinen jokin instanssi liikkuu, on tämä.
        https://en.wikipedia.org/wiki/Geodesics_in_general_relativity#Mathematical_expression
        'this equation is analogous to Newton's laws of motion'

        Sitä ei löytynyt vielä varsinaisena SR-aikana, mutta yhtälö ei vielä vaadi gravitaatiota koskevia väitteitä, ja siitä voi kirjoittaa äskeistä pyörimistä koskevan muodon (minkä lisäksi jos gravitaatiota vaatii, niin gravitaatiosta voi esittää periaatteen eli oletuksen, minkä jälkeen siitä saa lupauksena tämän yhtälön).

        Kun ihminen tai maapallo siis pyörii tai ovat paikallaan, kaikki havaitsijat pitävät tähtien liikettä tämän yhtälön mukaisina. Teoria lupaa, että voit ennustaa siitä jotain, kuten esim. missä suunnassa tähti lopulta on itsesi mielestä, ja voit myös kertoa, mitä se eri tavalla pyörivien muiden mielestä on jo ennen kuin nämä ovat sen kokeneet. Ja jotain tälle teorialle erikoisempaa voi ennustaa myös, kuten sen, että jos tähdestä lähtee valoa kohti toista tähteä, joka myös on kaukana ja paljon valoa nopeampi, niin silti valo ehtii tietyn matkan päähän lähemmäs toista tähteä, vaikka tähdet eivät itse koskaan saavuta toisiaan sillä, että pyörit. Ei ole mitään ristiriitaa tai tämän lupauksen pettämistä.

        ---

        Idea siitä, että universumi pyörii, ja sen nopeudet ovat hurjia, ei ole mikään Einsteinin vaihtoehto siltä osin, kuin mitä Einsteinista puhutaan havaitsijoihin ja näiden pyörimiseen liittyen. Siinäkin universumissa, mikä kiertää maapalloa, esiintyy ihmisiä, jotka tekevät piruetteja. Heidän haivaintonsa täytyy sovittaa paikaallaanolijoiden havaintoihin. Malli joka sen tekee, on jokin toinen suhteellisuus, joka ei ajatellut valonnopeutta rajana. Mutta teorian selitys tästä pyörimisestä on lähes täysin päällekkäinen Einsteinin selityksen kanssa

        2


      • Anonyymi
        Anonyymi kirjoitti:

        Focaultin heiluria ei voi selittää ilman gravitaatiota. Jos vaatisi paikalle toimivan Focault-heilurin, niin ei voisi olla varma siitä, että onko oikein arvostella pyöriviä havaitsijoita myös ilman gravitaatiota. Jolloin Einsteinin kritisointi on pienemmällä (kritiikin pätevyyden) alueella tehty kuin, mitä oli mahdollista. Onko se heiluri ollut tässä siksi, että henkilö joka haluaa kommentoida Einsteinia joutuu ensin todistamaan, että pyöriminen on olemassa eikä fiktiivistä (avaruudet ja koordinaatistot on tehty matemaattisesti ja on olemassa joku joka ei usko, että pyöriminen on olemassa vain sen takia, että se niissä esitetään)? Tämän pyörimisen epäfiktiivisyyden voi minusta todistaa sillä, että pörivän henkilön kasvoihin maalataan merkkejä tietyin välein. Ja kun nämä merkit liikkuvat ja toistuvat hänen katselijoidensa silmin nähtynä, niin se on riittävä todiste siitä, että hän pyörii (pyörimismatematiikasta saatujen tulosten mukaan).

        "Suhteellisuusteorian sisäinen ristiriita piilee siis siinä, että jos kaukainen tähti kiertää maapallon 24 tunnissa, niin sen ratanopeuden tulee ylittää monin kerroin valonnopeuden, mikä olisi vastoin teorian oletusta valonnopeudesta kosmisena rajanopeutena. Näin ollen Einsteinin suhteellisuusteoria lupaa enemmän kuin se pystyy lunastamaan, ja on suuri mysteeri, miksi tiedemaailma edelleen pitää sitä kiveen hakattuna paradigmana. Vallitseva tilanne muistuttaa ennemmin ideologiaa tai uskontoa kuin itselleen rehellistä tieteellistä maailmankuvaa."

        Lähellä oleva aurinko kiertää siis jaloillaan pyörivän ihmisen ympäri nopeammin kuin valo. Siitä saa 'hyvin sisäisen' eli matemaattista jargonia koskevan lauseen, että mikä nopeus ja mikä raja on oikeasti mitäkin. Eli pikemminkin kieliopillisia sääntöjä siitä, että mitä lauseita pitää esittää, jotta lauseet eivät olisi ristiriidassa minkään kanssa. Erikoisessa suhteellisuusteoriassa lukee tällaisen korjauksen jälkeen näin:
        https://en.wikipedia.org/wiki/Postulates_of_special_relativity
        'As measured in any inertial frame of reference, light is always propagated in empty space with a definite velocity c that is independent of the state of motion of the emitting body. Or: the speed of light in free space has the same value c in all inertial frames of reference.'

        Tämä on tässä oletus. Joten lause joka sanoo, että jotakin on luvattu, on mahdollisesti ristiriidassa kirjallisten lähteiden kanssa.

        Pyörivän ihmisen näkemä aurinko ei ole määritelmän mukainen inertiaalisessa koordinaatistossa liikkuva objekti. Tutkiessaan pyörivää ihmistä tulisi tehneeksi näin:
        https://physics.stackexchange.com/questions/154834/how-do-i-transform-onto-a-relativistic-rotating-frame-of-reference

        Teoriat olettavat varsinkin GR:ssä sen, että tähden liike on luonnonlakien mukainen vaikka ihmisen pää pyörisi miten tahansa. Koska kyseessä oli fysiikan uusi teoria, se myös lupasi kertoa, mikä on luonnonlaki, jota tässä sitten noudatetaan. Voidaan kuitenkin olla vielä montaa mieltä siitä, että onko luonnonlaki lupaus vai oletus.

        Einsteinin luonnolaki tähdille ja sille, miten myös valon yksittäinen jokin instanssi liikkuu, on tämä.
        https://en.wikipedia.org/wiki/Geodesics_in_general_relativity#Mathematical_expression
        'this equation is analogous to Newton's laws of motion'

        Sitä ei löytynyt vielä varsinaisena SR-aikana, mutta yhtälö ei vielä vaadi gravitaatiota koskevia väitteitä, ja siitä voi kirjoittaa äskeistä pyörimistä koskevan muodon (minkä lisäksi jos gravitaatiota vaatii, niin gravitaatiosta voi esittää periaatteen eli oletuksen, minkä jälkeen siitä saa lupauksena tämän yhtälön).

        Kun ihminen tai maapallo siis pyörii tai ovat paikallaan, kaikki havaitsijat pitävät tähtien liikettä tämän yhtälön mukaisina. Teoria lupaa, että voit ennustaa siitä jotain, kuten esim. missä suunnassa tähti lopulta on itsesi mielestä, ja voit myös kertoa, mitä se eri tavalla pyörivien muiden mielestä on jo ennen kuin nämä ovat sen kokeneet. Ja jotain tälle teorialle erikoisempaa voi ennustaa myös, kuten sen, että jos tähdestä lähtee valoa kohti toista tähteä, joka myös on kaukana ja paljon valoa nopeampi, niin silti valo ehtii tietyn matkan päähän lähemmäs toista tähteä, vaikka tähdet eivät itse koskaan saavuta toisiaan sillä, että pyörit. Ei ole mitään ristiriitaa tai tämän lupauksen pettämistä.

        ---

        Idea siitä, että universumi pyörii, ja sen nopeudet ovat hurjia, ei ole mikään Einsteinin vaihtoehto siltä osin, kuin mitä Einsteinista puhutaan havaitsijoihin ja näiden pyörimiseen liittyen. Siinäkin universumissa, mikä kiertää maapalloa, esiintyy ihmisiä, jotka tekevät piruetteja. Heidän haivaintonsa täytyy sovittaa paikaallaanolijoiden havaintoihin. Malli joka sen tekee, on jokin toinen suhteellisuus, joka ei ajatellut valonnopeutta rajana. Mutta teorian selitys tästä pyörimisestä on lähes täysin päällekkäinen Einsteinin selityksen kanssa

        2

        Todetakseen maapallon olevan pyörivä voi käyttää usein muuallakin kuin GR:ssä (jos GR:n seinien ulkopuolella olisi mitään muuta) mainittuja absoluuttisen pyörimisen testejä. Focaultin heiluri perustuu osittain näissä olevaan ilmiöön, ja sen yksinkertainen ilmenemismuoto on se, että jos pyörit tyhjässä avaruudessa ja vapautat itsestäsi jonkin esineen, johon ei vaikuta mikään vuorovaikutus mistään universumissa, niin näkisit sen alkavan kiertämään itseäsi. Normaalisti asiat kiertävät toisiaan ympäri vain, jos niiden välillä on vetovoima, mutta tämä esine ei pysty muodostamaan kaikkia kiertoratoja, mitä on voimien tapauksessa olemassa, vaan se voi ainoastaan näyttää ympyrältä tai spiraalilta joka pakenee luotasi, jos annat sille vahingossa liikkeen mihin tahansa suuntaan.

        Jos pyörimistä kokeilee pienoiskoossa ja se toimii näyttäen, että ihmisen on mahdollista absoluuttisesti pyöriä, niin voi olettaa, että se tarkoittaa myös maan voivan pyöriä. Tehdäkseen teorian, missä maa pysäytetään, tarvitaan teorioita voimista, jotka sen tekevät.

        Jos olettaa universumin kiertävän maata, täytyy väittää, että esim. raketin lähtiessä maasta sen täytyy suurimmassa osassa tapauksia liittyä universumiin ja olla kiertämättä maata lainkaan tähtien suhteen. Ei ole kuitenkaan löydettävissä mitään voimaa tai vaikutusta, joka houkuttelisi lähteneitä esineitä lopettamaan maan kanssa paikallaan olemista. Koska voidaan asettaa kappale mille tahansa kiertoradalle, joka on esim. vähän universumin kiertoliikettä hitaampi (esim. geosynkronisoitu), tai sitten monta kertaa universumin kiertonopeutta nopeampi kiertoliike, joka voikin olla miten lähellä maata tahansa miten nopea tahansa. Jokainen näistä liikkeistä jää melkein pysyvästi kappaleelle, sen sijaan että universumi tai maa yrittäisi vetää niitä tietynlaiseen kiertoon. Tämä tarkoittaisi etteivät tähdetkään ole koskaan pakotettu jostain kiertämään maapalloa, vaan kaiken olisi pitänyt syntyä siten.

        --

        Eräs ajatusongelma pyörimisestä tulee varmaan siitä, että kuvittelee liikaa Einsteinin esimerkkejä ja pitää niitä jotenkin teoreettisesti arvokkaina ja vaadittuina asioina. Eli että esim. on pakko pystyä lähettämään valoa kaukaa itselleen vaikka olisi pyörimässä, ja että koska esimerkit käyttävät valoa myös, niin tällaiseen pyörijään lähetetyn valon pitää antaa ensin samaa tietoa aikoihin liittyen, kuin mitä se tarjoaa paikallaan olijalle (*). Minkä jälkeen lisäksi pitäisi olla mahdollista jatkaa Einsteinin tekstien mukaisia työskentely- ja päättelyketjuja, missä teorian tulkinnat ajasta ja suhteellisuudesta esim. pitää tehdä sen valon perusteella (olettamalla valon olevan vakio, saa kaavan, mistä voi väittää liikkuvien asioiden aikojen olevan muutoksen alaisia). Tämä voi olla mahdollista vain, jos tekee valoesimerkistä tällöin suoraan GR-maailman (ilman gravitaatiota). Eli se on sellainen lopputulos, joka pyörimiseen sopii, ja mitä nyt väitettäisiin voitavan muodostaa vain valon ja ajan esimerkeistä. Mutta ei ole oletettavaa, että GR-teoriaa olisi mahdollista tehdä millään muulla tavalla kuin Einstein sen on tehnyt tai myöhemmät oppikirjat (etenkään äskeistä kaavaa ei tule eikä mitään kaavaa noin vain tule).

        (*) Pikemminkin on niin, että tieto tai pienen mahdollisuuden esiintyminen siitä, mitä aika luonteeltaan on, eli relativistista, on tuotu esiin Einsteinin tekstissä (josta tietystä tekstistä puhutaan vasta alla) siten, että minimoidaan tilannetta, jota tarkastellaan, ettei siinä olisi mitään muuta läsnä kuin aikakoordinaateilleen muutoksen tekevät liikkeet. Se mitä Einstein demonstroi ei ole sama kuin se, mihin Einstein yrittää palauttaa koko todellisuuden. Ja jos koko todellisuus kiinnostaa, niin Einstein on kertonut muualla, mitä sekin on.

        --

        3


      • Anonyymi
        Anonyymi kirjoitti:

        Todetakseen maapallon olevan pyörivä voi käyttää usein muuallakin kuin GR:ssä (jos GR:n seinien ulkopuolella olisi mitään muuta) mainittuja absoluuttisen pyörimisen testejä. Focaultin heiluri perustuu osittain näissä olevaan ilmiöön, ja sen yksinkertainen ilmenemismuoto on se, että jos pyörit tyhjässä avaruudessa ja vapautat itsestäsi jonkin esineen, johon ei vaikuta mikään vuorovaikutus mistään universumissa, niin näkisit sen alkavan kiertämään itseäsi. Normaalisti asiat kiertävät toisiaan ympäri vain, jos niiden välillä on vetovoima, mutta tämä esine ei pysty muodostamaan kaikkia kiertoratoja, mitä on voimien tapauksessa olemassa, vaan se voi ainoastaan näyttää ympyrältä tai spiraalilta joka pakenee luotasi, jos annat sille vahingossa liikkeen mihin tahansa suuntaan.

        Jos pyörimistä kokeilee pienoiskoossa ja se toimii näyttäen, että ihmisen on mahdollista absoluuttisesti pyöriä, niin voi olettaa, että se tarkoittaa myös maan voivan pyöriä. Tehdäkseen teorian, missä maa pysäytetään, tarvitaan teorioita voimista, jotka sen tekevät.

        Jos olettaa universumin kiertävän maata, täytyy väittää, että esim. raketin lähtiessä maasta sen täytyy suurimmassa osassa tapauksia liittyä universumiin ja olla kiertämättä maata lainkaan tähtien suhteen. Ei ole kuitenkaan löydettävissä mitään voimaa tai vaikutusta, joka houkuttelisi lähteneitä esineitä lopettamaan maan kanssa paikallaan olemista. Koska voidaan asettaa kappale mille tahansa kiertoradalle, joka on esim. vähän universumin kiertoliikettä hitaampi (esim. geosynkronisoitu), tai sitten monta kertaa universumin kiertonopeutta nopeampi kiertoliike, joka voikin olla miten lähellä maata tahansa miten nopea tahansa. Jokainen näistä liikkeistä jää melkein pysyvästi kappaleelle, sen sijaan että universumi tai maa yrittäisi vetää niitä tietynlaiseen kiertoon. Tämä tarkoittaisi etteivät tähdetkään ole koskaan pakotettu jostain kiertämään maapalloa, vaan kaiken olisi pitänyt syntyä siten.

        --

        Eräs ajatusongelma pyörimisestä tulee varmaan siitä, että kuvittelee liikaa Einsteinin esimerkkejä ja pitää niitä jotenkin teoreettisesti arvokkaina ja vaadittuina asioina. Eli että esim. on pakko pystyä lähettämään valoa kaukaa itselleen vaikka olisi pyörimässä, ja että koska esimerkit käyttävät valoa myös, niin tällaiseen pyörijään lähetetyn valon pitää antaa ensin samaa tietoa aikoihin liittyen, kuin mitä se tarjoaa paikallaan olijalle (*). Minkä jälkeen lisäksi pitäisi olla mahdollista jatkaa Einsteinin tekstien mukaisia työskentely- ja päättelyketjuja, missä teorian tulkinnat ajasta ja suhteellisuudesta esim. pitää tehdä sen valon perusteella (olettamalla valon olevan vakio, saa kaavan, mistä voi väittää liikkuvien asioiden aikojen olevan muutoksen alaisia). Tämä voi olla mahdollista vain, jos tekee valoesimerkistä tällöin suoraan GR-maailman (ilman gravitaatiota). Eli se on sellainen lopputulos, joka pyörimiseen sopii, ja mitä nyt väitettäisiin voitavan muodostaa vain valon ja ajan esimerkeistä. Mutta ei ole oletettavaa, että GR-teoriaa olisi mahdollista tehdä millään muulla tavalla kuin Einstein sen on tehnyt tai myöhemmät oppikirjat (etenkään äskeistä kaavaa ei tule eikä mitään kaavaa noin vain tule).

        (*) Pikemminkin on niin, että tieto tai pienen mahdollisuuden esiintyminen siitä, mitä aika luonteeltaan on, eli relativistista, on tuotu esiin Einsteinin tekstissä (josta tietystä tekstistä puhutaan vasta alla) siten, että minimoidaan tilannetta, jota tarkastellaan, ettei siinä olisi mitään muuta läsnä kuin aikakoordinaateilleen muutoksen tekevät liikkeet. Se mitä Einstein demonstroi ei ole sama kuin se, mihin Einstein yrittää palauttaa koko todellisuuden. Ja jos koko todellisuus kiinnostaa, niin Einstein on kertonut muualla, mitä sekin on.

        --

        3

        https://youtu.be/thZn1O-aXPw
        Himankan videolla 9:00:
        Tässä tilanteessa, joka on esitelty ei ole läheskään samoja tapahtumia kuin mitä muiden kirjoittajien kelloparadokseissa on (monien kelloparadoksi on sama kuin kaksoisparadoksi). Jos tulet erottamaan jonkun kellon, tulet erottamaan myös kuka liikkuu mitenkin. Jos erotat liikkeeksi jotain, et ala erottamaan liikkeeksi mitään muuta kuin sen mitä erotit. Samoin erotat vain yhden kellon per liike. Sille jonka olet jo erottanut liikkuvan nopeimmin tulet erottamaan hitaimman kellon. Esimerkiksi erotat sen että et itse liiku, vaan muut likkuvat, jolloin muilla on hitaampi kello (1 kpl). Tosiasiassa ei ole kuitenkaan olemassa mitään toisten luona avaruudessa tai myöskään toisissa nopeuksissa (sinun lävitsesi menemässä) olevien aikojen mittauksia. Jokaisessa väitteessä, missä puhutaan useiden objektien kelloista, on oltava hyvä perustelu, miksi joku tuntee ne. Tai sitten kyseessä on vain matemaattinen aikamuuttujien pyörittely paperilla (missä ajatellaan, että en ole enää kelloni vaan olen toinen kello). Tästäkin voitaisiin sanoa, että ei paperilla kirjoitetusta tule tulokseksi mitään ristiriitaa, kuten 1 = 2. Periaatteessa filosofit kai haluaisivat olevan niin, että väärä teoria tekisi itsestään tällaisen tuloksen, joka ei voi olla oikein, mutta ei sellaista tule suhteellisuusteorioissa mistään eteen. Miksei mahdoton asia sitten osoitu mahdottomuudeksi myös kun sitä laskee?

        Jos kaksi autoa ajaisi satasta toisian vastaan tiellä, jota he eivät saa nähdä, ja jos heillä olisi konepelti, joka on tasan metrin mittainen ja jokin ajanmittausväline, niin he kumpikin voisivat mitata toisen auton nopeudeksi 200. Huom. kun keskustellaan tästä asiasta ja esitetään dioja siitä, niin kumpikin auto on nopeampi kuin toinen auto. Tarvittiin vain Newtonin fysiikka huomaamaan, että meille on valehdeltu nopeudesta, mutta miksi Newtonin teoria ei ole mahdotonta, ja miksi autolla ajaminen näkemättä tietä ei ole mahdotonta?

        9:30
        Tässä on sanottu, että valonnopeuden ja jonkin X:n suhteellisuuden välillä on ristiriita. Filosofisesti tämän ristiriidan yksi lopullinen poisto on esim. se, että luonnonlakeja ei ole olemassa ja kaikkien havainnot ovat toisiinsa liittymättömiä kokemuksia. Mutta ilman sitä voidaan sanoa myös, että vastaava ristiriita Galilein avaruudessa (johon SR:stä palattaisiin) olisi se, että avaruus on absoluuttinen, mutta nopeus ei ole. Tämä on vielä vahvempi analogia SR:n valolle, kun sanotaan, että SR:ssä absoluuttista on Minkowskin aikaavaruusintervallin pituus valolle. Galilei tulisi sisältämään aina myös absoluuttisia avaruusintervalleja. Ja sitten siinä toimivat tutkijat koettaisivat soveltaa eri havaitsijoiden toisistaan poikeeavia havaintoja jotenkin tähän faktaan, vaikka filosofisesti heidän tekemisensä ei pysty homogenisoitumaan absoluuttiseksi tai suhteelliseksi arvostelijoiden mielestä.

        18:00:
        Astronomical Journal, vol. 81, page 452, 1976. Varmaan jotain mitä Hawking oli nähnyt, mutta hän voi tarkoittaa myös muita 1920-luvun kokeita, joista ei syntynyt samanlaista epäilyä kuin Eddingtonista. Jotkut muut kuin Hawking ovat väittäneet, että Eddingtonin esittämät tulokset oli saatu hyvin ja jo he saivat väittää olevansa GR:ää puoltava testi:
        https://arxiv.org/ftp/arxiv/papers/0709/0709.0685.pdf
        'Nevertheless, as I have argued above, I believe that close inspection of the totality of
        information now available to us since 1979, suggests that the 1919 experimenters
        probably were justified in concluding that they had at least falsified the lower Newtonian
        prediction.'

        Haluavatko filosofit ottaa tuomarin aseman siinä, että onko Hawking vai nämä muut oikeassa ja saisimmeko seurata tätä päätöksentekoa kauttaaltaan? (Tosin ihan ensimmäisenä Hawking itse ei laittanut missään kirjassa tikkua ristiin sen puolesta, että olisi Eddingtonia arvioinut, vaan hän toistaa tuon ajan yleistä mielipidettä.) (Äskeinen kirjoituskin teoretisoi Hawkingin kirjan syntyä.)

        18:30
        Ivesin kokeet eivät kumoa SR:ää, mutta kumoavat kaiken, missä ei ole SR:n efektiä ajalle. Tämä on edelleen väitetysti kokeista kertomista, mutta Himanka keskustelee tässä vain siitä, että mikä on yhden ihmisen mielipide. Ivesin mukaan kokessa näkyvä efekti on Lorentz-eetteriksi tulkittava. Jos Ives ei tiennyt, että se on tulkinta (koska Einsteinin piti mennä sanomalehteen sanomaan, että tuo todistaa hänen SR:nsä) ja että efektit tämän ja SR:n välillä ovat samoja, niin voidaan mielummin sanoa, että hänen mielipiteelläänkään SR:stä ei ole mitään väliä, koska hän ei ole vielä tähän mennessä tutustunut siihen. Ja mielipiteisiinkin tulee ainakin epäilyksen varjoja, että ovatko ne lopulta lukittuja.

        4


      • Anonyymi
        Anonyymi kirjoitti:

        https://youtu.be/thZn1O-aXPw
        Himankan videolla 9:00:
        Tässä tilanteessa, joka on esitelty ei ole läheskään samoja tapahtumia kuin mitä muiden kirjoittajien kelloparadokseissa on (monien kelloparadoksi on sama kuin kaksoisparadoksi). Jos tulet erottamaan jonkun kellon, tulet erottamaan myös kuka liikkuu mitenkin. Jos erotat liikkeeksi jotain, et ala erottamaan liikkeeksi mitään muuta kuin sen mitä erotit. Samoin erotat vain yhden kellon per liike. Sille jonka olet jo erottanut liikkuvan nopeimmin tulet erottamaan hitaimman kellon. Esimerkiksi erotat sen että et itse liiku, vaan muut likkuvat, jolloin muilla on hitaampi kello (1 kpl). Tosiasiassa ei ole kuitenkaan olemassa mitään toisten luona avaruudessa tai myöskään toisissa nopeuksissa (sinun lävitsesi menemässä) olevien aikojen mittauksia. Jokaisessa väitteessä, missä puhutaan useiden objektien kelloista, on oltava hyvä perustelu, miksi joku tuntee ne. Tai sitten kyseessä on vain matemaattinen aikamuuttujien pyörittely paperilla (missä ajatellaan, että en ole enää kelloni vaan olen toinen kello). Tästäkin voitaisiin sanoa, että ei paperilla kirjoitetusta tule tulokseksi mitään ristiriitaa, kuten 1 = 2. Periaatteessa filosofit kai haluaisivat olevan niin, että väärä teoria tekisi itsestään tällaisen tuloksen, joka ei voi olla oikein, mutta ei sellaista tule suhteellisuusteorioissa mistään eteen. Miksei mahdoton asia sitten osoitu mahdottomuudeksi myös kun sitä laskee?

        Jos kaksi autoa ajaisi satasta toisian vastaan tiellä, jota he eivät saa nähdä, ja jos heillä olisi konepelti, joka on tasan metrin mittainen ja jokin ajanmittausväline, niin he kumpikin voisivat mitata toisen auton nopeudeksi 200. Huom. kun keskustellaan tästä asiasta ja esitetään dioja siitä, niin kumpikin auto on nopeampi kuin toinen auto. Tarvittiin vain Newtonin fysiikka huomaamaan, että meille on valehdeltu nopeudesta, mutta miksi Newtonin teoria ei ole mahdotonta, ja miksi autolla ajaminen näkemättä tietä ei ole mahdotonta?

        9:30
        Tässä on sanottu, että valonnopeuden ja jonkin X:n suhteellisuuden välillä on ristiriita. Filosofisesti tämän ristiriidan yksi lopullinen poisto on esim. se, että luonnonlakeja ei ole olemassa ja kaikkien havainnot ovat toisiinsa liittymättömiä kokemuksia. Mutta ilman sitä voidaan sanoa myös, että vastaava ristiriita Galilein avaruudessa (johon SR:stä palattaisiin) olisi se, että avaruus on absoluuttinen, mutta nopeus ei ole. Tämä on vielä vahvempi analogia SR:n valolle, kun sanotaan, että SR:ssä absoluuttista on Minkowskin aikaavaruusintervallin pituus valolle. Galilei tulisi sisältämään aina myös absoluuttisia avaruusintervalleja. Ja sitten siinä toimivat tutkijat koettaisivat soveltaa eri havaitsijoiden toisistaan poikeeavia havaintoja jotenkin tähän faktaan, vaikka filosofisesti heidän tekemisensä ei pysty homogenisoitumaan absoluuttiseksi tai suhteelliseksi arvostelijoiden mielestä.

        18:00:
        Astronomical Journal, vol. 81, page 452, 1976. Varmaan jotain mitä Hawking oli nähnyt, mutta hän voi tarkoittaa myös muita 1920-luvun kokeita, joista ei syntynyt samanlaista epäilyä kuin Eddingtonista. Jotkut muut kuin Hawking ovat väittäneet, että Eddingtonin esittämät tulokset oli saatu hyvin ja jo he saivat väittää olevansa GR:ää puoltava testi:
        https://arxiv.org/ftp/arxiv/papers/0709/0709.0685.pdf
        'Nevertheless, as I have argued above, I believe that close inspection of the totality of
        information now available to us since 1979, suggests that the 1919 experimenters
        probably were justified in concluding that they had at least falsified the lower Newtonian
        prediction.'

        Haluavatko filosofit ottaa tuomarin aseman siinä, että onko Hawking vai nämä muut oikeassa ja saisimmeko seurata tätä päätöksentekoa kauttaaltaan? (Tosin ihan ensimmäisenä Hawking itse ei laittanut missään kirjassa tikkua ristiin sen puolesta, että olisi Eddingtonia arvioinut, vaan hän toistaa tuon ajan yleistä mielipidettä.) (Äskeinen kirjoituskin teoretisoi Hawkingin kirjan syntyä.)

        18:30
        Ivesin kokeet eivät kumoa SR:ää, mutta kumoavat kaiken, missä ei ole SR:n efektiä ajalle. Tämä on edelleen väitetysti kokeista kertomista, mutta Himanka keskustelee tässä vain siitä, että mikä on yhden ihmisen mielipide. Ivesin mukaan kokessa näkyvä efekti on Lorentz-eetteriksi tulkittava. Jos Ives ei tiennyt, että se on tulkinta (koska Einsteinin piti mennä sanomalehteen sanomaan, että tuo todistaa hänen SR:nsä) ja että efektit tämän ja SR:n välillä ovat samoja, niin voidaan mielummin sanoa, että hänen mielipiteelläänkään SR:stä ei ole mitään väliä, koska hän ei ole vielä tähän mennessä tutustunut siihen. Ja mielipiteisiinkin tulee ainakin epäilyksen varjoja, että ovatko ne lopulta lukittuja.

        4

        19:26:
        Tässä mainitun kokeen tilanne ja käytäntö on aivan eri kuin diassa 9:00. Jos jokin on kelloparadoksin empiirinen duplikaatti (sanoja jotka parhaiten kuvaavat sitä, että ottaa teoreettisen tilanteen nimeltä 'kelloparadoksi' ja kokeilee sitä empiirisesti), niin kyseessä ei ole kelloparadoksin ratkaisu. Kelloparadoksin voi ratkaista vain lopettamalla Newtonisen kuvittelun. Minkä jälkeen saman teoreettisen tilanteen kuin kelloparadoksi nimi on vain 'yksi tilanteista'. Äskeisessä tarkoitan sitäkin, että kelloparadoksin duplikoiminen empiiriseen kokeeseen on paradoksin ymmärtämisen tai selittämisen kannalta aivan yhtä hyvä kuin duplikoisi kelloparadoksin paperilla toisenlaiseen tehtävänantoon tai esim. vähän laajempaan laskutehtävään, missä on useampia kappaleita tai useampia nopeuksia, tai jos sen vain ottaisi Xeroxilla ensimmäisestä paradoksista.

        Kohdassa 20:00 on jonkinlaista outoa narratiivia, että joku ei tiedä, mikä liiikkuu sen jälkeen, kun jotain on valittu paikallaan pysyväksi. Mutta että jos tietää, että eetteri ei eetterin mielestä liiku, niin silloin tietää mikä liikkuu? SR:ssä voidaan sanoa, että valitaan ns. tähtien antama koordinaatisto, missä kaikki liikkuu pois paikaltaan vähintään maapallon tahdissa (näin Hafele ja Keating ovat kerran esitelleet). Ei tässä ole mitään eetteriin johtavaa. Auttaisiko tässä tietää, että SR:ssä näiden liikkuvien kellojen mallin kuvauksessa ei ole väliä, minkä kellon sanoo alussa olevan se, mitä ei liikutetta, mutta että se mitä tästä ennustetaan tällä tavalla, on aivan yhtä oikea tulos, kuin jos valitsisi toisin. Kun pitää valinnastaan kiinni loppuun asti niin tästä tulee ennustetta, ja ilmeisesti on totta, että tämä on ollut koe johon ennustetta joku vertaa. Tällöin jos ennustus ja koe on sama niin kyseessä on oikeaa ennustetta. Vaatii vielä vähän enemmän eri koordinaatistoista nähtyjen näkökulmien ymmärtämistä, jos haluaa tietää enemmän niistä kaikista yhtäaikaa. Tai jos jopa haluaa tietää, että maailma on objektiivinen ja lainkuuliainen paikka.

        Jotkut tehtävänannot ja selostukset voivat olla Lorentzin mukaisia vs. Einsteinin mukaisia, jos niissä lukee, että täällä on etteri joka menee näin ja näin. Kokeentekijätkin joutuvat joskus kirjoittamaan kokeesta näin, jos heidän pitäisi selittää, mikä kokeen tapahtuma merkitsisi mitä tapahtumaa teoriassa, ja miten koe tietää varautua siihen, että tietoa juuri teoriasta tulisi sitä kautta ilmi (se kenen juttuja koetetaan tuoda ilmi tai falsifioida saa varmaan nimensä esiin eniten ja kuten yllä sanotaan niin samat asiat falsifioivat SR:ää ja Lorentz-eetteriä). Ei silti ole mitään Lorentzin koe-asetelmaa, ellei ole olemassa Lorentz-huonetta, Lorentz-valoilla, Lorenz-putkea, Lorentz-ainetta,Lorentz-taulua Lorentz-viisareilla.

        'Ota kello ja kulje' ei ole mitään selittelyjä vaativa koe-koe, mutta tekijät näkivät itse sen vaivan, että kirjoittivat näkyviin, mitä 'teoria' ennustaa aikaerojen olevan.
        https://paulba.no/paper/Hafele_Keating.pdf
        Lisäksi paperi selittää miksi otettiin kolme kelloa, vaikka paradoksissa ei ole kolmea. Ja mietittiin virheitä. Missää näistäkään ei viitattu Lorentzin ajatteluun. Esimerkiksi kellojen virheet, joista puhutaan, ovat vriheitä, joita voi tarkastella kellon lepokoordinaatistossa pysytellen. Ja itseasiassa on aivan päinvastoin kuin Himanka sanoo: Hafele-Keating koe oli liikkuvien objektien lisäksi gravitaatiovaikutuksen koe. Lorentzin-eetterissä ei ole gravitaatiota, ja Lorentzin malli ei ole vastannut Hafele-Keating -tyyppisen kokeen tuloksia edes ensimmäisessä yrityksessä saavutetulla tarkkuudella.
        https://en.wikipedia.org/wiki/Hafele%E2%80%93Keating_experiment#Results

        Välillä mainittu Einstein-Lorentz -teoria tarkoitaa sitä, että SR on joidenkin mielestä yli 50 % Lorentzin teoria. Einstein-Lorentz -teoria ei sisällä eetteriä ja epärelativismia ellei sitä suoraan mainita.

        22:30
        Tämä salainen koodi satelliiteissa ei todista mitään, jos se on salainen. Lisäksi se ei todista mitään julkisenakaan siksi, että kun äskeiseen vertaa, niin kuka tahansa pystyy tekemään vaihtoehtoisen gravitaation, joka toimii niin pitkälle samalla tavalla kuin GR, että GPS tulee tarpeeksi lähelle eli niissä on molemmissa yhtä monta matemaattista termiä, kuin koodi nykyään käyttää. Sen jälkeen SR/GR-koodin yhtä osaa voi sanoa Lorentziksi ja toista uudeksi gravitaatioksi.

        GR:n kumoamista varten pitää tehdä koe, joka mittaa GR:n ennustuksen onnistumista eikä salailla koodeja. Salailu ei myöskään auta mitään, jos yritettäisiin olla falsifioitumatta jossain toisessa teoriassa kuin GR. Jos olisi fakta, että mikään GPS-koodi ei ole koskaan ollut GR-koodia, niin se tarkoittaisi, että ei ole ymmärretty GR:n kumoamisen käsitettä. Tai että Hatch on jopa Einsteinin lähettämä vakooja.

        5


      • Anonyymi
        Anonyymi kirjoitti:

        19:26:
        Tässä mainitun kokeen tilanne ja käytäntö on aivan eri kuin diassa 9:00. Jos jokin on kelloparadoksin empiirinen duplikaatti (sanoja jotka parhaiten kuvaavat sitä, että ottaa teoreettisen tilanteen nimeltä 'kelloparadoksi' ja kokeilee sitä empiirisesti), niin kyseessä ei ole kelloparadoksin ratkaisu. Kelloparadoksin voi ratkaista vain lopettamalla Newtonisen kuvittelun. Minkä jälkeen saman teoreettisen tilanteen kuin kelloparadoksi nimi on vain 'yksi tilanteista'. Äskeisessä tarkoitan sitäkin, että kelloparadoksin duplikoiminen empiiriseen kokeeseen on paradoksin ymmärtämisen tai selittämisen kannalta aivan yhtä hyvä kuin duplikoisi kelloparadoksin paperilla toisenlaiseen tehtävänantoon tai esim. vähän laajempaan laskutehtävään, missä on useampia kappaleita tai useampia nopeuksia, tai jos sen vain ottaisi Xeroxilla ensimmäisestä paradoksista.

        Kohdassa 20:00 on jonkinlaista outoa narratiivia, että joku ei tiedä, mikä liiikkuu sen jälkeen, kun jotain on valittu paikallaan pysyväksi. Mutta että jos tietää, että eetteri ei eetterin mielestä liiku, niin silloin tietää mikä liikkuu? SR:ssä voidaan sanoa, että valitaan ns. tähtien antama koordinaatisto, missä kaikki liikkuu pois paikaltaan vähintään maapallon tahdissa (näin Hafele ja Keating ovat kerran esitelleet). Ei tässä ole mitään eetteriin johtavaa. Auttaisiko tässä tietää, että SR:ssä näiden liikkuvien kellojen mallin kuvauksessa ei ole väliä, minkä kellon sanoo alussa olevan se, mitä ei liikutetta, mutta että se mitä tästä ennustetaan tällä tavalla, on aivan yhtä oikea tulos, kuin jos valitsisi toisin. Kun pitää valinnastaan kiinni loppuun asti niin tästä tulee ennustetta, ja ilmeisesti on totta, että tämä on ollut koe johon ennustetta joku vertaa. Tällöin jos ennustus ja koe on sama niin kyseessä on oikeaa ennustetta. Vaatii vielä vähän enemmän eri koordinaatistoista nähtyjen näkökulmien ymmärtämistä, jos haluaa tietää enemmän niistä kaikista yhtäaikaa. Tai jos jopa haluaa tietää, että maailma on objektiivinen ja lainkuuliainen paikka.

        Jotkut tehtävänannot ja selostukset voivat olla Lorentzin mukaisia vs. Einsteinin mukaisia, jos niissä lukee, että täällä on etteri joka menee näin ja näin. Kokeentekijätkin joutuvat joskus kirjoittamaan kokeesta näin, jos heidän pitäisi selittää, mikä kokeen tapahtuma merkitsisi mitä tapahtumaa teoriassa, ja miten koe tietää varautua siihen, että tietoa juuri teoriasta tulisi sitä kautta ilmi (se kenen juttuja koetetaan tuoda ilmi tai falsifioida saa varmaan nimensä esiin eniten ja kuten yllä sanotaan niin samat asiat falsifioivat SR:ää ja Lorentz-eetteriä). Ei silti ole mitään Lorentzin koe-asetelmaa, ellei ole olemassa Lorentz-huonetta, Lorentz-valoilla, Lorenz-putkea, Lorentz-ainetta,Lorentz-taulua Lorentz-viisareilla.

        'Ota kello ja kulje' ei ole mitään selittelyjä vaativa koe-koe, mutta tekijät näkivät itse sen vaivan, että kirjoittivat näkyviin, mitä 'teoria' ennustaa aikaerojen olevan.
        https://paulba.no/paper/Hafele_Keating.pdf
        Lisäksi paperi selittää miksi otettiin kolme kelloa, vaikka paradoksissa ei ole kolmea. Ja mietittiin virheitä. Missää näistäkään ei viitattu Lorentzin ajatteluun. Esimerkiksi kellojen virheet, joista puhutaan, ovat vriheitä, joita voi tarkastella kellon lepokoordinaatistossa pysytellen. Ja itseasiassa on aivan päinvastoin kuin Himanka sanoo: Hafele-Keating koe oli liikkuvien objektien lisäksi gravitaatiovaikutuksen koe. Lorentzin-eetterissä ei ole gravitaatiota, ja Lorentzin malli ei ole vastannut Hafele-Keating -tyyppisen kokeen tuloksia edes ensimmäisessä yrityksessä saavutetulla tarkkuudella.
        https://en.wikipedia.org/wiki/Hafele%E2%80%93Keating_experiment#Results

        Välillä mainittu Einstein-Lorentz -teoria tarkoitaa sitä, että SR on joidenkin mielestä yli 50 % Lorentzin teoria. Einstein-Lorentz -teoria ei sisällä eetteriä ja epärelativismia ellei sitä suoraan mainita.

        22:30
        Tämä salainen koodi satelliiteissa ei todista mitään, jos se on salainen. Lisäksi se ei todista mitään julkisenakaan siksi, että kun äskeiseen vertaa, niin kuka tahansa pystyy tekemään vaihtoehtoisen gravitaation, joka toimii niin pitkälle samalla tavalla kuin GR, että GPS tulee tarpeeksi lähelle eli niissä on molemmissa yhtä monta matemaattista termiä, kuin koodi nykyään käyttää. Sen jälkeen SR/GR-koodin yhtä osaa voi sanoa Lorentziksi ja toista uudeksi gravitaatioksi.

        GR:n kumoamista varten pitää tehdä koe, joka mittaa GR:n ennustuksen onnistumista eikä salailla koodeja. Salailu ei myöskään auta mitään, jos yritettäisiin olla falsifioitumatta jossain toisessa teoriassa kuin GR. Jos olisi fakta, että mikään GPS-koodi ei ole koskaan ollut GR-koodia, niin se tarkoittaisi, että ei ole ymmärretty GR:n kumoamisen käsitettä. Tai että Hatch on jopa Einsteinin lähettämä vakooja.

        5

        23:23
        Tässä diassa olevassa tekstissä on monta ongelmaa filosofiselta kannalta. Mikä kaikki X voi selittää sitä, että Y ei ole olemassa? Jos Y olisi liitettävissä GR:ään, niin miksi Y:tä ei lopulta ole, ja mitä tekemistä sillä on X:n kanssa jos Y:tä ei ole? Jos tämä dia olisi tieteellinen ja tosiasioihin perustuva, niin miksi asioista välittäminen olisi jotain 50 %:sta välittämistä asioista?

        Fysiikan kannalta tässä on ehkä kirjoitusvirheenä mainittu gradientti. Suurimmat ajan muutokset gravitaatiosta voidaan approksimoida gravitaation potentiaalin avulla. Ajan muutos on sellaisena suureena annettu, että sen suureen suurin muutos ei ole kuitenkaan verrannollinen potentiaalin gradienttiin vaan potentiaalin arvoon. Joten tässä tekstissä ei voi mitenkään olettaa olevan totta edes, että GR olisi taho, joka olisi ennustanut jotain siten, että tahtoi laittaa jonkin objektin potentiaalin gradientin johonkin. Tavalliset ihmiset olisivat myös sellaisia, kuin Himanka viitaa kohdassa 44:40, että he pystyisivät sovittamaan jonkin kaavan tuloksiin, mutta he eivät mitään tekemättä olisi tietoisia siitä, että kyseessä on näkymättömän entiteetin eli potentiaalin nimenomaan vielä gradientti. GR:n teoria kahden kappaleen mallista on täynnä kaikkea, mutta gradientti siltikin pysyy vain tietyn arvoisena puskuna kyseiseen suuntaan, eli jos ei ole aika-gradienttia, mikä on sama kuin ajasta riippuva gravitaation vahvuus yhdellä etäisyydellä esim. maasta, niin mitään ei viedä toisenlaiseen kellonkäyntiin. Siinäkin aikaero on vain syntynyt, kun summataan ajanjaksoja samoin kuin sanoisi, että miten paljon nopeammassa liikkeessä on nopeasti kiihdyttävä kappale.

        GR:n lopputulos GPS-ajalle auringon avulla laskettuna on tietääkseni liian pieni, jotta sitä olisi ehdotettu testattavaksi. Tietämättä miten eetteristi tässä on aikonut tuoda gravitaation eli maapallon efektin satelliittiin, niin ei ole varmaa voisiko, sanoa että jos maa olisi lähempänä aurinkoa, niin sitten vertailu olisi tapahtunut. Periaatteessa jos eetteristi alkaa säveltää esim. maakohtaista eetteriä, niin sama keksintö tulee missä tahansa vaiheessa kyseeseen, kun ongelma siirretään esim. aurinkoon (joko vain siihen tai jopa kahden kappaleen järjestelmään, joka GR:n mielestä voisi olla mikä tahansa tarkoittaen, että nykyään labarotoriogravitaatiot voitasiin toteuttaa sellaisina, että eetteristi voisi selittää, mikä on hänestä oikea kahden kappaleen eetteri).

        31:30
        Einsteinin vastauksessa on hyvin otettu huomioon se, että pyöriminen ei ole mikään ainoa tapaus, missä kappaleiden nopeudet voivat olla käsittämättömän paljon valoa nopeampia. Lisäksi siinä lukee, että jos on kuullut Einsteinin väittävän, että pyöriessä tulisi näkyä korkeintaan valonnopeuksia, niin tämä ei lainkaan vastaa sitä, mitä henkilön piti kuulla.

        35:40
        Dia on erittäin sekava eikä puhe sitä selvennä kovasti. Himanka syöksyy kysymksestä nimeltä 'eetteri vai SR' johonkin 'ongelmaan'. Hän laittaa ""-merkit ratkaisuun, ja minä laitan ne ongelmaan, Aluksi siksi, että esitys on niin epäjohdonmukainen. Jotkut ongelmavaihtoehdot, mitä tästä diasta voisi kehittää olisivat myös sellaisia, että niitä nimittäisi jälkeenpäin "ongelmiksi". SItä mitä esim. Einstein on tekemässä ei pohjusteta mitenkään eli ei kerrota, mikä on 'samanaikaisuuden määritelmä' ja miksi se on tähän diaan valittu. Samanaikaisuuden määritelmä (mikä on vain Einsteinia) on mielummin ratkaisu kuin ongelma, koska silloin sen arvostelulla on suurempi merkitys. Mutta tietääkö kukaan, mikä on ongelma, mihin samanaikaisuuden määritelmä on ratkaisu?

        Himankalla on tässä diassa ilmeisesti seuraavanlaisia päämääriä, menetelmiä ja ajatuskulkuja. Hän ajattelee, että jos hän löytää teorian teksistä kohdan, missä lukee 'jokin on vapaa päätös', silloin hän tekee tästä päätelmän, että jokainen voi tehdä vapaan päätöksen siitä, että uskooko teoriaan vai ei. Hän tekee tätä myös sitä varten, että kohta hän sanoo olevansa tietoinen siitä, että Lorentz-eetteri on toinen SR tulkinta. Himanka on ehkä ajatellut, että tämä on perusteltava jotenkin, ja hän kuvittelee perustelun olevan se, että jossain kohtaa SR:ää lukee, että jokin asia on vapaa päätös. Siten diaan on otettu myös toinen kappale, joka on outo torso, mutta sisältää sen arvokkaan jalokiven, että jossain lukee jotain tällaisina vapaussanoina.

        6


      • Anonyymi
        Anonyymi kirjoitti:

        23:23
        Tässä diassa olevassa tekstissä on monta ongelmaa filosofiselta kannalta. Mikä kaikki X voi selittää sitä, että Y ei ole olemassa? Jos Y olisi liitettävissä GR:ään, niin miksi Y:tä ei lopulta ole, ja mitä tekemistä sillä on X:n kanssa jos Y:tä ei ole? Jos tämä dia olisi tieteellinen ja tosiasioihin perustuva, niin miksi asioista välittäminen olisi jotain 50 %:sta välittämistä asioista?

        Fysiikan kannalta tässä on ehkä kirjoitusvirheenä mainittu gradientti. Suurimmat ajan muutokset gravitaatiosta voidaan approksimoida gravitaation potentiaalin avulla. Ajan muutos on sellaisena suureena annettu, että sen suureen suurin muutos ei ole kuitenkaan verrannollinen potentiaalin gradienttiin vaan potentiaalin arvoon. Joten tässä tekstissä ei voi mitenkään olettaa olevan totta edes, että GR olisi taho, joka olisi ennustanut jotain siten, että tahtoi laittaa jonkin objektin potentiaalin gradientin johonkin. Tavalliset ihmiset olisivat myös sellaisia, kuin Himanka viitaa kohdassa 44:40, että he pystyisivät sovittamaan jonkin kaavan tuloksiin, mutta he eivät mitään tekemättä olisi tietoisia siitä, että kyseessä on näkymättömän entiteetin eli potentiaalin nimenomaan vielä gradientti. GR:n teoria kahden kappaleen mallista on täynnä kaikkea, mutta gradientti siltikin pysyy vain tietyn arvoisena puskuna kyseiseen suuntaan, eli jos ei ole aika-gradienttia, mikä on sama kuin ajasta riippuva gravitaation vahvuus yhdellä etäisyydellä esim. maasta, niin mitään ei viedä toisenlaiseen kellonkäyntiin. Siinäkin aikaero on vain syntynyt, kun summataan ajanjaksoja samoin kuin sanoisi, että miten paljon nopeammassa liikkeessä on nopeasti kiihdyttävä kappale.

        GR:n lopputulos GPS-ajalle auringon avulla laskettuna on tietääkseni liian pieni, jotta sitä olisi ehdotettu testattavaksi. Tietämättä miten eetteristi tässä on aikonut tuoda gravitaation eli maapallon efektin satelliittiin, niin ei ole varmaa voisiko, sanoa että jos maa olisi lähempänä aurinkoa, niin sitten vertailu olisi tapahtunut. Periaatteessa jos eetteristi alkaa säveltää esim. maakohtaista eetteriä, niin sama keksintö tulee missä tahansa vaiheessa kyseeseen, kun ongelma siirretään esim. aurinkoon (joko vain siihen tai jopa kahden kappaleen järjestelmään, joka GR:n mielestä voisi olla mikä tahansa tarkoittaen, että nykyään labarotoriogravitaatiot voitasiin toteuttaa sellaisina, että eetteristi voisi selittää, mikä on hänestä oikea kahden kappaleen eetteri).

        31:30
        Einsteinin vastauksessa on hyvin otettu huomioon se, että pyöriminen ei ole mikään ainoa tapaus, missä kappaleiden nopeudet voivat olla käsittämättömän paljon valoa nopeampia. Lisäksi siinä lukee, että jos on kuullut Einsteinin väittävän, että pyöriessä tulisi näkyä korkeintaan valonnopeuksia, niin tämä ei lainkaan vastaa sitä, mitä henkilön piti kuulla.

        35:40
        Dia on erittäin sekava eikä puhe sitä selvennä kovasti. Himanka syöksyy kysymksestä nimeltä 'eetteri vai SR' johonkin 'ongelmaan'. Hän laittaa ""-merkit ratkaisuun, ja minä laitan ne ongelmaan, Aluksi siksi, että esitys on niin epäjohdonmukainen. Jotkut ongelmavaihtoehdot, mitä tästä diasta voisi kehittää olisivat myös sellaisia, että niitä nimittäisi jälkeenpäin "ongelmiksi". SItä mitä esim. Einstein on tekemässä ei pohjusteta mitenkään eli ei kerrota, mikä on 'samanaikaisuuden määritelmä' ja miksi se on tähän diaan valittu. Samanaikaisuuden määritelmä (mikä on vain Einsteinia) on mielummin ratkaisu kuin ongelma, koska silloin sen arvostelulla on suurempi merkitys. Mutta tietääkö kukaan, mikä on ongelma, mihin samanaikaisuuden määritelmä on ratkaisu?

        Himankalla on tässä diassa ilmeisesti seuraavanlaisia päämääriä, menetelmiä ja ajatuskulkuja. Hän ajattelee, että jos hän löytää teorian teksistä kohdan, missä lukee 'jokin on vapaa päätös', silloin hän tekee tästä päätelmän, että jokainen voi tehdä vapaan päätöksen siitä, että uskooko teoriaan vai ei. Hän tekee tätä myös sitä varten, että kohta hän sanoo olevansa tietoinen siitä, että Lorentz-eetteri on toinen SR tulkinta. Himanka on ehkä ajatellut, että tämä on perusteltava jotenkin, ja hän kuvittelee perustelun olevan se, että jossain kohtaa SR:ää lukee, että jokin asia on vapaa päätös. Siten diaan on otettu myös toinen kappale, joka on outo torso, mutta sisältää sen arvokkaan jalokiven, että jossain lukee jotain tällaisina vapaussanoina.

        6

        https://en.wikisource.org/wiki/Relativity_(1931)/Section_8
        Jos dian ongelma on nimeltään kehäpäättelyn ongelma, niin silloin ongelma mistä puhutaan Einsteinin kirjassa ei ole SR-ongelma eikä Lorentz-ongelma, vaan se on yhden Einsteinin esittämän ajatusleikin muodostama ongelma, missä joku satunnainen henkilö, jolla ei ole mitään teoriaa, joutuisi selvittämään mitä on varma tieto samanaikaisuudesta kahdelle muka samaan aikaan nähdylle salamaniskulle, joiden välissä hän ensin päättää olevansa. Einstein sanoo, että jos joutuisi itse ottamaan selvää siitä, että tuleeko molemmista salamista valoa samalla nopeudella, niin tähän nopeuden tarkastamiseen tarvittaisiin ajan määrittely, joka menisi edelle sitä tietoa, jota ajasta tähän mennessä on. Mihin aikaa tässä tarkistamisessa tarvitaan, ei ole kerrottu erikseen, joten joudut tietämään sen itse, jos asetut kannattamaan tämän ongelman konkreettisuutta. Ongelma ei koske tässä pelkästään valoa eikä sellaista valoa, joka on vakio kaikkien eri nopeuksisien havaitsijoiden mielestä. Valon olemassaolon olettamisesta on ollut valtavasti hyötyä siinä, että aikatietoa voisi alkaa pyytämään varmennetussa muodossa. Jos uskottaisiin pelkästään yksittäiseen Newtonin aikaan tai eetterin aikaan (salamapisteet ja keskipiste ovat eetterissä paikoillaan), niin voitaisiin kysyä sama kysymys kuin Einstein kysyy, että mistä saadaan varma tieto sille, että onko toinen salama samanaikainen tapahtuma kuin toinen. Mikään teoria ei siis ole tekstissä mainitusta kehäpäättelystä vapautettu. Lorentzin ainoa vastaus tähän periaatteessa on se, että 'en voi saada mitään tietoa siitä, enkä halua saada, koska uskon vain absoluuttiseen aikaan enkä siihen että ajalle koituisi seurauksia siitä, että siitä esitetään moisia kysymyksiä; ja missä minulla kuitenkin sattuu olemaan myös sellainen valon teoria käytössä, missä se näyttää kulkevan sopivasti vakionopeudella'.

        Äskeinen tilanne esiintyisi samanlaisena myös yhdessä ulottuvuudessa, jos määriteltäisiin etäisyys mittatikun avulla, mutta jonkun mielikuvitus on sellainen, että hän ei pidä mittatikkuja varmana tietona, vaan ajattelee niiden voivan olla jonkin vaikutuksen alla, missä tikkujen pituus muuttuu, kun ne ovat kauempana keskeltä ja kahdessa eri paikassa. Nopeutta ja edes liikettä ei ole olemassa yllä tai kenenkään teoriassa ilman aikaa ja paikkaa, ja samalla ajan mittausta ei ole olemassa 'mittatikuilla ajan suunnassa'. Tämä on sellainen tiedon raja, joka filosofienkin pitäisi ottaa realismina.

        Kun Einstein sanoo, että valon kulusta tehty ajatus ei ole fysikaalinen, niin tämä ei ole minusta täysin kirjaimellista kaikissa sanan merkityksissä, vaan se on ainoastaan viittaus siihen, että mikä on tällä hetkellä empiirisesti varmistettavissa. Kun havainnot eivät estä tekemästä teoriaa, että valo tekee näin, niin sitä oletusta saa käyttää ja tehdä siitä päätelmiä. Ja kohta ylipäätään tarkoittaa sitä, että saa arvata ja väittää mitä tahansa ja kokeilla sitä empiirisesti. Siinä on ajateltu sellaista tieteenfilosofiaa, joka on peräisin tieteilijästä eikä filosofista.

        Äskeiseen liittyen moderni Minkowskilainen SR (varsinkin kun se on vain yksi GR:n osa-alue) on näiden läheteltyjen valojen asemasta sellainen oletusketju, että siinä oletetaan jokin aika-avaruus. Tämän jälkeen erilaisten aika-avaruuksien oletuksesta seuraa ennusteita kokeita varten. Tämä on osaltaan sellaista matematiikkalähtöistä tiedettä, mitä Einstein lanseerasi maailmalle. Kysymyksiä mitä tähän voisi ehkä kuvittella liitettävän on se, että 'onko aika-avaruus määritelmä' ja mikä on aika-avaruus määritelmän suhde kokeelliseen (nyt tätä jälkimmäistä kysymystä on pohtinut vain Einstein tapauksessa, joka sanoo, että Euklidinen aika-avaruus tuntuu olevan epävarmasti kokeiltavissa ja samalla tavalla mahdollisesti kaikki muutkin).

        En muuten löytänyt Lorentzin kirjettä, ja olisin halunnut sen siksi, että siitä voisi lukea, mikä on Lorentzin oikea mielentila hänen puhuessaan vapauksista. Yksi halpa argumenttini Himankan päämäärähakuista ajatusjuoksua kohtaan on se, että esitän Lorentzin teoriasta kohdan, missä Lorentz tekee jotain, mikä ei koske luontoa ja on vain päätetöksen perusteella jotain, mitä voi käyttää. Tämä on
        https://en.wikipedia.org/wiki/Lorentz_ether_theory#Local_time
        Väitän tässä että apumuuttujana käytetty lokaali aika on Lorentzin omien ajatusten mukaan pelkkää mielikuvituksen käyttöä. Hänen tapauksessaan valitaan käyttää mielikuvitusta tai sitten ei valita. Valitessa tämän ajan saa käyttää Lorentz-kovariantteja luonnonlakeja, mutta jos ei valitse, niin luonnolakeja ei varsinaisesti ole tai ne voivat tuntua siltä, että niitä valitaan poimintoina. Himanka ei ole varsinaisesti selvitellyt, mitä (mistä) Einstein valitsee käyttää. Tässä on pari vaihtoehtoa, jotka esiintyvät seuraavissa kohdissa.

        7


      • Anonyymi
        Anonyymi kirjoitti:

        https://en.wikisource.org/wiki/Relativity_(1931)/Section_8
        Jos dian ongelma on nimeltään kehäpäättelyn ongelma, niin silloin ongelma mistä puhutaan Einsteinin kirjassa ei ole SR-ongelma eikä Lorentz-ongelma, vaan se on yhden Einsteinin esittämän ajatusleikin muodostama ongelma, missä joku satunnainen henkilö, jolla ei ole mitään teoriaa, joutuisi selvittämään mitä on varma tieto samanaikaisuudesta kahdelle muka samaan aikaan nähdylle salamaniskulle, joiden välissä hän ensin päättää olevansa. Einstein sanoo, että jos joutuisi itse ottamaan selvää siitä, että tuleeko molemmista salamista valoa samalla nopeudella, niin tähän nopeuden tarkastamiseen tarvittaisiin ajan määrittely, joka menisi edelle sitä tietoa, jota ajasta tähän mennessä on. Mihin aikaa tässä tarkistamisessa tarvitaan, ei ole kerrottu erikseen, joten joudut tietämään sen itse, jos asetut kannattamaan tämän ongelman konkreettisuutta. Ongelma ei koske tässä pelkästään valoa eikä sellaista valoa, joka on vakio kaikkien eri nopeuksisien havaitsijoiden mielestä. Valon olemassaolon olettamisesta on ollut valtavasti hyötyä siinä, että aikatietoa voisi alkaa pyytämään varmennetussa muodossa. Jos uskottaisiin pelkästään yksittäiseen Newtonin aikaan tai eetterin aikaan (salamapisteet ja keskipiste ovat eetterissä paikoillaan), niin voitaisiin kysyä sama kysymys kuin Einstein kysyy, että mistä saadaan varma tieto sille, että onko toinen salama samanaikainen tapahtuma kuin toinen. Mikään teoria ei siis ole tekstissä mainitusta kehäpäättelystä vapautettu. Lorentzin ainoa vastaus tähän periaatteessa on se, että 'en voi saada mitään tietoa siitä, enkä halua saada, koska uskon vain absoluuttiseen aikaan enkä siihen että ajalle koituisi seurauksia siitä, että siitä esitetään moisia kysymyksiä; ja missä minulla kuitenkin sattuu olemaan myös sellainen valon teoria käytössä, missä se näyttää kulkevan sopivasti vakionopeudella'.

        Äskeinen tilanne esiintyisi samanlaisena myös yhdessä ulottuvuudessa, jos määriteltäisiin etäisyys mittatikun avulla, mutta jonkun mielikuvitus on sellainen, että hän ei pidä mittatikkuja varmana tietona, vaan ajattelee niiden voivan olla jonkin vaikutuksen alla, missä tikkujen pituus muuttuu, kun ne ovat kauempana keskeltä ja kahdessa eri paikassa. Nopeutta ja edes liikettä ei ole olemassa yllä tai kenenkään teoriassa ilman aikaa ja paikkaa, ja samalla ajan mittausta ei ole olemassa 'mittatikuilla ajan suunnassa'. Tämä on sellainen tiedon raja, joka filosofienkin pitäisi ottaa realismina.

        Kun Einstein sanoo, että valon kulusta tehty ajatus ei ole fysikaalinen, niin tämä ei ole minusta täysin kirjaimellista kaikissa sanan merkityksissä, vaan se on ainoastaan viittaus siihen, että mikä on tällä hetkellä empiirisesti varmistettavissa. Kun havainnot eivät estä tekemästä teoriaa, että valo tekee näin, niin sitä oletusta saa käyttää ja tehdä siitä päätelmiä. Ja kohta ylipäätään tarkoittaa sitä, että saa arvata ja väittää mitä tahansa ja kokeilla sitä empiirisesti. Siinä on ajateltu sellaista tieteenfilosofiaa, joka on peräisin tieteilijästä eikä filosofista.

        Äskeiseen liittyen moderni Minkowskilainen SR (varsinkin kun se on vain yksi GR:n osa-alue) on näiden läheteltyjen valojen asemasta sellainen oletusketju, että siinä oletetaan jokin aika-avaruus. Tämän jälkeen erilaisten aika-avaruuksien oletuksesta seuraa ennusteita kokeita varten. Tämä on osaltaan sellaista matematiikkalähtöistä tiedettä, mitä Einstein lanseerasi maailmalle. Kysymyksiä mitä tähän voisi ehkä kuvittella liitettävän on se, että 'onko aika-avaruus määritelmä' ja mikä on aika-avaruus määritelmän suhde kokeelliseen (nyt tätä jälkimmäistä kysymystä on pohtinut vain Einstein tapauksessa, joka sanoo, että Euklidinen aika-avaruus tuntuu olevan epävarmasti kokeiltavissa ja samalla tavalla mahdollisesti kaikki muutkin).

        En muuten löytänyt Lorentzin kirjettä, ja olisin halunnut sen siksi, että siitä voisi lukea, mikä on Lorentzin oikea mielentila hänen puhuessaan vapauksista. Yksi halpa argumenttini Himankan päämäärähakuista ajatusjuoksua kohtaan on se, että esitän Lorentzin teoriasta kohdan, missä Lorentz tekee jotain, mikä ei koske luontoa ja on vain päätetöksen perusteella jotain, mitä voi käyttää. Tämä on
        https://en.wikipedia.org/wiki/Lorentz_ether_theory#Local_time
        Väitän tässä että apumuuttujana käytetty lokaali aika on Lorentzin omien ajatusten mukaan pelkkää mielikuvituksen käyttöä. Hänen tapauksessaan valitaan käyttää mielikuvitusta tai sitten ei valita. Valitessa tämän ajan saa käyttää Lorentz-kovariantteja luonnonlakeja, mutta jos ei valitse, niin luonnolakeja ei varsinaisesti ole tai ne voivat tuntua siltä, että niitä valitaan poimintoina. Himanka ei ole varsinaisesti selvitellyt, mitä (mistä) Einstein valitsee käyttää. Tässä on pari vaihtoehtoa, jotka esiintyvät seuraavissa kohdissa.

        7

        Osoitan että vapaa harkinta -lause teorian keskellä ei salli tehdä mitään johtopäätöksiä teoriasta suhteessa toisiin teorioihin. SR on kuin sellainen teoria, missä lukee
        SR on lause R
        R on lauseet a ja b ja c ja (d tai e) ja f...
        Voidaan valita d tai e

        Tästä ei tietenkään seuraa, että SR on aivan sama asia kuin jokin teoria, missä on
        Lause E
        E on lauseet alpha, beta, gamma, delta...

        Lisäksi nimenomaan tiedetään, että Lorentzin-eetterissä ei ole lausetta (d tai e) = S. Merkitään sitä vielä S:llä.

        Mitä d ja e merkitsevät tässä on sama kuin, että Einstein valitsi yksisuuntaisen valonnopeuden olevan sama, kun se tulee mistä hyvänsä. Jos sanoo, että tämän voi valita toisin, niin puhutaan 'samanaikaisuuden määritelmän konventionaalisuudesta'. Fyysikot eivät melkein koskaan puhu tästä, vaan ajattelevat, että joku on joskus tehnyt teorian, jossa on valittu juuri tämä valo, ja nyt sitä teoriaa saa käyttää kunnes se tekee virheen. Ja että tämä on nimeltään tiedettä. Voit katsoa tästä kuitenkin, mitä seuraa jos kuvittelee, että joku on vastuussa siitä, että mielikuvitus voi luoda muita vaihtoehtoja:
        https://sites.pitt.edu/~jdnorton/teaching/HPS_0410/chapters/significance_conv_sim/index.html

        Samanaikauisuuden määritelmä, joka tekee tai ainakin näyttää sen, mitä on ajan suhteellisuus, on kuin lause S yllä. Voidaan sanoa, että tämä on seurauksiltaan sellainen, että se on teoreettisesti juuri eri asia kuin Lorentzin eetterin koordinaatiston absoluuttinen aika. Yksi asia, mitä Einstein saavuttaa SR:llä on kuin se, että sen jälkeen voi sanoa, että eetterin pitäminen fyysisesti todellisena ei ole enää pakollista, vaan se on valinta. Oikea filosofi olisi verrannut valinnanvaraisuuksissa sitä, miten paljon ne merkitsevät. Einsteinin tulos, joka on saavutettu uuden ajattelun tulos, on sama kuin sanottaisiin, että tähän asti fyysikot (paitsi jo esim. Poincare) ajattelivat, että he ovat huoneessa, jossa huoneella on ominaisuuksia. Mihin SR sanoi, että te olette valinneet olevanne huoneessa. SR silti jättää osan ominaisuuksista olemaan vaikka huone otetaan pois, missä tarkoitan, että Maxwellin yhtälöt ja jokin mikä niitä noudattaa on eetterin ominaisuus, mutta että sillä oli aina muitakin ominaisuuksia, jotka tekivät siitä kuin huoneen. Tietämys ja filosofia tässä toimii lisäksi siten, että SR:n jälkeen ei voida kelata ajattelua koskaan taakse päin tilanteeseen, missä voisi sanoa, että Lorentz-eetterin tekemä huone on normaali ehdotus, joka on varmaa tietoa jos se on oikeassa.

        Eli yksi asia mitä Einstein valitsee, on se että hän valitsee olla käyttämättä huonetta. Tämä valinnanvapaus ei ole kuitenkaan kaikki, vaan lisäksi voidaan esittää kysymys, että onko fysiikassa oikea tapa edetä se, että jos ympärillä oleva huone on valinta, niin sitä ei olisi pakko mainita?

        39:00
        Jos tulkinnasta on pakko siirtyä toiseen tulkintaan, niin sillä hetkellä ne ovat lakanneet olemasta vain tulkintoja. Olisi ollut parempi löytää oikeasti neutraaleja lähdeviitteitä, jos haluaa neutraaliutta korostaa. Tai sitten sanoa videon hetkellä 0:00, että 'kuten kaikki hyvin tietävät...', ...niin nämä ovat tulkintoja. Minkä jälkeen loppuvideo olisikin ollut esitelmä eri aikoina olleista fanaatikoista, jotka eivät tätä käsitettä hyväksy.

        ---

        Kun videolla puhuttiin rotaatiosta, siinä puuttuu se kohta, missä sanotaan, mitä Lorentz esittää pyörivästä havaitsijasta eetterissä (vs. paikallaan oleva havaitsija eetterissä joka pyörii). Miksi kukaan on viehtynyt siihen, että Maxwellin yhtälöt lakkaavat tällöin toimimasta (puhuttaisiin liikkuvasta valoaallosta eikä tähdistä)? Tai mitä filosofisempaa eroa on pyörijää kiertävällä valon hiukkasella ja eetterin hiukkasella, jota tämä havaitsija pitää ympärillään kiertävänä, mutta joka Lorentzin mukaan ei liikuta sähkömagneettista tietoa valoa nopeammin ollessaan valon väliaine?

        ---

        8


      • Anonyymi
        Anonyymi kirjoitti:

        Osoitan että vapaa harkinta -lause teorian keskellä ei salli tehdä mitään johtopäätöksiä teoriasta suhteessa toisiin teorioihin. SR on kuin sellainen teoria, missä lukee
        SR on lause R
        R on lauseet a ja b ja c ja (d tai e) ja f...
        Voidaan valita d tai e

        Tästä ei tietenkään seuraa, että SR on aivan sama asia kuin jokin teoria, missä on
        Lause E
        E on lauseet alpha, beta, gamma, delta...

        Lisäksi nimenomaan tiedetään, että Lorentzin-eetterissä ei ole lausetta (d tai e) = S. Merkitään sitä vielä S:llä.

        Mitä d ja e merkitsevät tässä on sama kuin, että Einstein valitsi yksisuuntaisen valonnopeuden olevan sama, kun se tulee mistä hyvänsä. Jos sanoo, että tämän voi valita toisin, niin puhutaan 'samanaikaisuuden määritelmän konventionaalisuudesta'. Fyysikot eivät melkein koskaan puhu tästä, vaan ajattelevat, että joku on joskus tehnyt teorian, jossa on valittu juuri tämä valo, ja nyt sitä teoriaa saa käyttää kunnes se tekee virheen. Ja että tämä on nimeltään tiedettä. Voit katsoa tästä kuitenkin, mitä seuraa jos kuvittelee, että joku on vastuussa siitä, että mielikuvitus voi luoda muita vaihtoehtoja:
        https://sites.pitt.edu/~jdnorton/teaching/HPS_0410/chapters/significance_conv_sim/index.html

        Samanaikauisuuden määritelmä, joka tekee tai ainakin näyttää sen, mitä on ajan suhteellisuus, on kuin lause S yllä. Voidaan sanoa, että tämä on seurauksiltaan sellainen, että se on teoreettisesti juuri eri asia kuin Lorentzin eetterin koordinaatiston absoluuttinen aika. Yksi asia, mitä Einstein saavuttaa SR:llä on kuin se, että sen jälkeen voi sanoa, että eetterin pitäminen fyysisesti todellisena ei ole enää pakollista, vaan se on valinta. Oikea filosofi olisi verrannut valinnanvaraisuuksissa sitä, miten paljon ne merkitsevät. Einsteinin tulos, joka on saavutettu uuden ajattelun tulos, on sama kuin sanottaisiin, että tähän asti fyysikot (paitsi jo esim. Poincare) ajattelivat, että he ovat huoneessa, jossa huoneella on ominaisuuksia. Mihin SR sanoi, että te olette valinneet olevanne huoneessa. SR silti jättää osan ominaisuuksista olemaan vaikka huone otetaan pois, missä tarkoitan, että Maxwellin yhtälöt ja jokin mikä niitä noudattaa on eetterin ominaisuus, mutta että sillä oli aina muitakin ominaisuuksia, jotka tekivät siitä kuin huoneen. Tietämys ja filosofia tässä toimii lisäksi siten, että SR:n jälkeen ei voida kelata ajattelua koskaan taakse päin tilanteeseen, missä voisi sanoa, että Lorentz-eetterin tekemä huone on normaali ehdotus, joka on varmaa tietoa jos se on oikeassa.

        Eli yksi asia mitä Einstein valitsee, on se että hän valitsee olla käyttämättä huonetta. Tämä valinnanvapaus ei ole kuitenkaan kaikki, vaan lisäksi voidaan esittää kysymys, että onko fysiikassa oikea tapa edetä se, että jos ympärillä oleva huone on valinta, niin sitä ei olisi pakko mainita?

        39:00
        Jos tulkinnasta on pakko siirtyä toiseen tulkintaan, niin sillä hetkellä ne ovat lakanneet olemasta vain tulkintoja. Olisi ollut parempi löytää oikeasti neutraaleja lähdeviitteitä, jos haluaa neutraaliutta korostaa. Tai sitten sanoa videon hetkellä 0:00, että 'kuten kaikki hyvin tietävät...', ...niin nämä ovat tulkintoja. Minkä jälkeen loppuvideo olisikin ollut esitelmä eri aikoina olleista fanaatikoista, jotka eivät tätä käsitettä hyväksy.

        ---

        Kun videolla puhuttiin rotaatiosta, siinä puuttuu se kohta, missä sanotaan, mitä Lorentz esittää pyörivästä havaitsijasta eetterissä (vs. paikallaan oleva havaitsija eetterissä joka pyörii). Miksi kukaan on viehtynyt siihen, että Maxwellin yhtälöt lakkaavat tällöin toimimasta (puhuttaisiin liikkuvasta valoaallosta eikä tähdistä)? Tai mitä filosofisempaa eroa on pyörijää kiertävällä valon hiukkasella ja eetterin hiukkasella, jota tämä havaitsija pitää ympärillään kiertävänä, mutta joka Lorentzin mukaan ei liikuta sähkömagneettista tietoa valoa nopeammin ollessaan valon väliaine?

        ---

        8

        "Syksy Räsänen kehoitti blogissaan seuraamaan aeon.co kosmologian uutisia. Siellä on useitakin artikkeleita pimeän aineen vaihtoehdosta. Tässä esim. New Yorkin yliopiston fysiikan professori selittää MOND-teoriasta, miten pimeää ainetta ei todennäköisesti ole olemassa vaan MOND on paljon parempi selitys:
        https://aeon.co/essays/we-should-explore-alternatives-to-the-standard-model-of-cosmology
        Pimeää ainetta + suhtista voidaan siis pitää falsifioituna, eivät päde galaksien mittakaavassa."

        Räsäsen blogit ja kommentit ovat olleet ykskantaan jonkin yleistetyn pimeän aineen puolella, kuten
        https://www.ursa.fi/blogi/kosmokseen-kirjoitettua/viidakon-lait/
        Hänellä on omia kommenttejaan MOND:ista eivätkä ne näytä positiivisilta. On outoa, miten hänet pystyi liittämään noinkin moneen epäpimeäaine tekstiin kuin tuolla aeon-sivustolla on, mutta hän ei maininnut aeonin olevan juuri sen kosmologian esseiden lukemista varten, vaan sen koko science-osio mainittiin kommenteissa vanhemmassa blogissa, joka sekin arvioi kriittisesti julkista mediaa kaikissa tiedeaiheissa. Ja kaikkiin tiedeaiheisiin kysyttiin hyviä lehtiä kommenteissa. Hän on toistuvasti ehdottanut lukemistoon myös tässä olevaa quantamagazinea, joka on lähempänä journalistien aikakausilehteä kuin tuo. Jos juuri MOND on tulossa hyväksi tai huonoksi, niin näyttäisi, että nimenomaan Räsäsenkin esseiden tai häneltä kysymisen kautta siitä kuulisi nopeammin kuin aeonista, joka laahaa niin monta vuotta myöhässä, kuin miten vanhoja tekstit ovat, ja lisättynä niin paljon kuin niiden kirjoittajat laahaavat kehitystä jäljessä.

        9


      • Anonyymi
        Anonyymi kirjoitti:

        "Syksy Räsänen kehoitti blogissaan seuraamaan aeon.co kosmologian uutisia. Siellä on useitakin artikkeleita pimeän aineen vaihtoehdosta. Tässä esim. New Yorkin yliopiston fysiikan professori selittää MOND-teoriasta, miten pimeää ainetta ei todennäköisesti ole olemassa vaan MOND on paljon parempi selitys:
        https://aeon.co/essays/we-should-explore-alternatives-to-the-standard-model-of-cosmology
        Pimeää ainetta suhtista voidaan siis pitää falsifioituna, eivät päde galaksien mittakaavassa."

        Räsäsen blogit ja kommentit ovat olleet ykskantaan jonkin yleistetyn pimeän aineen puolella, kuten
        https://www.ursa.fi/blogi/kosmokseen-kirjoitettua/viidakon-lait/
        Hänellä on omia kommenttejaan MOND:ista eivätkä ne näytä positiivisilta. On outoa, miten hänet pystyi liittämään noinkin moneen epäpimeäaine tekstiin kuin tuolla aeon-sivustolla on, mutta hän ei maininnut aeonin olevan juuri sen kosmologian esseiden lukemista varten, vaan sen koko science-osio mainittiin kommenteissa vanhemmassa blogissa, joka sekin arvioi kriittisesti julkista mediaa kaikissa tiedeaiheissa. Ja kaikkiin tiedeaiheisiin kysyttiin hyviä lehtiä kommenteissa. Hän on toistuvasti ehdottanut lukemistoon myös tässä olevaa quantamagazinea, joka on lähempänä journalistien aikakausilehteä kuin tuo. Jos juuri MOND on tulossa hyväksi tai huonoksi, niin näyttäisi, että nimenomaan Räsäsenkin esseiden tai häneltä kysymisen kautta siitä kuulisi nopeammin kuin aeonista, joka laahaa niin monta vuotta myöhässä, kuin miten vanhoja tekstit ovat, ja lisättynä niin paljon kuin niiden kirjoittajat laahaavat kehitystä jäljessä.

        9

        Wiki: "Ad hoc on latinaa ja tarkoittaa ”tätä (tarkoitusta, tehtävää) varten”. Tieteessä ja filosofiassa ad hoc -hypoteesi tarkoittaa hypoteesia, joka hyväksytään puhtaasti teorian pelastamiseksi vaikeuksilta tai kumoamiselta, ilman mitään itsenäistä perustetta.[1] "

        Syksy Räsänen ilmeisesti ei tiedä mitä ad hoc tarkoittaa, mutta esim. pimeä aine on hyvä esimerkki ad hoc -selityksestä. Pimeän aineen avulla yritetään pelastaa GR-hypoteesia, joten muuten voisi pitää ilman muuta falsifioituna.

        Pimeään aineeseen pätee kaikki ad hoc -hypoteesin tunnusmerkit. Ei ole esim. havaittu ensimmäistäkään pimeän aineen hiukkasta eikä tiedetä mistä ne voisivat koostua. Kaikki selitykset kuten MACHOt, WIMPit, aksionit jne sen sijaan on kokeellisesti falsifioitu.


      • Anonyymi
        Anonyymi kirjoitti:

        Wiki: "Ad hoc on latinaa ja tarkoittaa ”tätä (tarkoitusta, tehtävää) varten”. Tieteessä ja filosofiassa ad hoc -hypoteesi tarkoittaa hypoteesia, joka hyväksytään puhtaasti teorian pelastamiseksi vaikeuksilta tai kumoamiselta, ilman mitään itsenäistä perustetta.[1] "

        Syksy Räsänen ilmeisesti ei tiedä mitä ad hoc tarkoittaa, mutta esim. pimeä aine on hyvä esimerkki ad hoc -selityksestä. Pimeän aineen avulla yritetään pelastaa GR-hypoteesia, joten muuten voisi pitää ilman muuta falsifioituna.

        Pimeään aineeseen pätee kaikki ad hoc -hypoteesin tunnusmerkit. Ei ole esim. havaittu ensimmäistäkään pimeän aineen hiukkasta eikä tiedetä mistä ne voisivat koostua. Kaikki selitykset kuten MACHOt, WIMPit, aksionit jne sen sijaan on kokeellisesti falsifioitu.

        Uuden tutkimuksen (simulaation) mukaan galaksin keskusta toimii gravitaatiolinssinä, joka taivuttaa galaksin toiselta puolelta tulevia gravitaatioaaltoja, ja tällä tavoin saa aikaiseksi voimakkaamman gravitaatiovaikutuksen kauempana keskustasta oleviin tähtiin. Ei siis tarvita sen enempää pimeää ainetta kuin mond -teoriaakaan. Uusi MOGA-teoria selittää havainnot täydellisesti ja ilman mystisiä voimia ja aineita.
        https://arxiv.org/abs/2403.02848


      • Anonyymi
        Anonyymi kirjoitti:

        Uuden tutkimuksen (simulaation) mukaan galaksin keskusta toimii gravitaatiolinssinä, joka taivuttaa galaksin toiselta puolelta tulevia gravitaatioaaltoja, ja tällä tavoin saa aikaiseksi voimakkaamman gravitaatiovaikutuksen kauempana keskustasta oleviin tähtiin. Ei siis tarvita sen enempää pimeää ainetta kuin mond -teoriaakaan. Uusi MOGA-teoria selittää havainnot täydellisesti ja ilman mystisiä voimia ja aineita.
        https://arxiv.org/abs/2403.02848

        Pimeällä aineella yritetään siis pelastaa virheelliset laskelmat. GR sinänsä ei ole ollut väärässä, mutta kosmologit eivät osaa laskea oikein.


    • Anonyymi

      Melko tarkka matemaattinen malli, ja syyksi toimivuuteen on nimetty hypoteettinen aika-avaruus yhdessä painovoiman kanssa - joka teorian mukaan syntyy kappaleiden aiheuttamasta aika-avaruuden kaareutumisesta.

    • Anonyymi

      Jaaha, Kouko Nieminen heilahtelupyörähtelynsä kanssa on saanut seuraajan.

    • Anonyymi

      Filosofi Himanka ei nyt kyllä millään tavalla perustele, miten Lorenzin eetteriteoria olisi Einsteinin suhteellisuusteoriaa parempi. Saa jopa sellainen käsityksen, ettei herra filosofi kunnolla ymmärrä kumpaakaan mallia.

      Eetteriteoriassa on siis yksi erityinen lepokoordinaatisto, joka vastaa kaikkialla läsnäolevaa etteeriä. Todellinen ja absoluuttinen aika on se tahti, jota kyseisessä lepokoordinaatistossa oleva kello käy. Lisäksi valo kulkee vakionopeudella eetterin suhteen. Tämä eetteri on kuitenkin puhtaasti matemaattinen rakenne, jota ei voi havaita. Filosofisesti erittäin huono lähtökohta!

      Lisäksi Lorenz oletti, että muissa lepokoordinaatistoissa kuin erityisessä eetterin lepokoordinaatistossa liikkuvat kappaleet kutistuvat Lorenzin muunnoksen mukaisesti. Siis hokkuspokkustempulla oletetaan muunnos, joka voidaan Einsteinin suhteellisuusteoriassa johtaa fysikaalisista lähtökohdista. Itse en näe mitään syytä rummuttaa eetteriteorian puolesta, mikä vaikuttaa väkinäiseltä yritykseltä sovittaa newtonilainen aikakäsitys yhteen pituuskontraktion kanssa.

      • Anonyymi

        No jos Himanka ei erota ympyräliikettä suoraviivaisesta liikkeestä, niin turha häneltä on vaatia ymmärrystä eetteri- ja suhteellisuusteorian yksityiskohdista.


    • Anonyymi

      Suhtiksen ei tietenkään anneta kumoutua. Sitä puolustetaan niin kuin BB:täkin. Se on kuin uskonto, jonka korkein on virheetön ja sitä virheettömyyttä puolustetaan. Kuten Einstein sanoi, "jos faktat ei tue teoriaa, vaihda faktat". Teoria, eli tieteellinen uskomus, on siis tärkeimmässä asemassa. Suhtis ei siis kumoudu, koska epäsopivat faktat hylätään ja sallitaan vain ne, jotka tukee suhtista.

      • Anonyymi

        No annappa meille esimerkkejä niistä "epäsopivista faktoista", jotka eivät tue suhteellisuusteoriaa? Mutta jos länkytät samaa lapsenmielistä paskaa kuin Himanka, niin voit antaa olla.


      • Anonyymi

        Einstein ei koskaan sanonut noin. Ehkä tää Himanka on voinut sanoa :D


    • Anonyymi

      😍😋😍😋😍😋😍😋😍

      😋 K­­u­u­­­m­­a­t­ ­­­t­­­y­­­t­­­ö­­­t­­ ­­­o­­d­o­t­t­a­v­­­­­­a­­t­­ ­­s­­i­­n­­­u­­­a­ -> https://us4.fun/kissgirl?18310007

      🔞❤️❤️❤️❤️❤️🔞💋💋💋💋💋🔞

    Ketjusta on poistettu 1 sääntöjenvastaista viestiä.

    Luetuimmat keskustelut

    1. Istuva kansanedustaja epäiltynä pahoinpitelystä ja ampuma-aserikoksesta

      Seiskan tietojen mukaan Timo Vornanen on epäiltynä pahoinpitelystä ja ampuma-aserikoksesta eikä kenellekään taatusti tul
      Maailman menoa
      432
      3115
    2. Timo Vornanen kilahti

      Mikähän sille kansanedustajalle polisiisi miehelle on noin pahasti mennyt hermot , että tulevaisuudensa pilasi totaalise
      Kotka
      107
      2502
    3. Tollokin tajuaa että Timo Vornanen

      oli joutunut äärimmäiseen tilanteeseen ampuessaa yhden laukauksen katuun. Ei poliisi tee tuollaista hetken mielijohteest
      Maailman menoa
      368
      2326
    4. Pullonpalautusjärjestelmä muuttuu - paluu menneisyyteen

      EU suuressa viisaudessaan on päättänyt, että pulloja pitää kierrättää. Jos oikein ymmärsin, nykyisen järjestelmänmme ti
      Maailman menoa
      158
      2035
    5. Sininen farmari - Ford Focus- YFB-842 on poliisilta kadoksissa Kauhajärvellä

      https://alibi.fi/uutiset/poliisilta-poikkeuksellinen-vihjepyynto-autossa-oleva-henkilo-on-avuntarpeessa/?shared=29255-2d
      Lapua
      7
      1857
    6. Onko oikeudenmukaista? Yhdellä taholla yllättävä valta-asema Tähdet, tähdet -voittajan valinnassa!

      Näinpä, onko sinusta tämä oikein? Viime jaksossakin voittaja selvisi vain yhden äänen erolla ja tänä sunnuntaina ensimm
      Tv-sarjat
      23
      1277
    7. 166
      1265
    8. No kerros nyt nainen

      Kumpi mielestäsi oli se joka väärinkäsitti kaiken? Nyt voi olla jo rehellinen kun koko tilanne on jo lähes haihtunut.
      Ikävä
      97
      1176
    9. Persukansanedustaja Timo Vornanen ammuskellut Helsingissä

      Poliisi siviiliammatiltaan, luvallinen ase mukana baarissa tällä hemmetin valopääpersulla. Meni eduskunnasta suoraan baa
      Haapavesi
      72
      1111
    10. Nainen, mietit miten minä jaksan

      En voi hyvin. Nykyään elämäni on lähinnä selviytymistä tunnista ja päivästä toiseen. Usein tulee epävarma olo, että mite
      Ikävä
      88
      967
    Aihe